You are on page 1of 50

Manual CTO

de Medicina y Ciruga

9. edicin

Urologa

Autora
Sara Daz Naranjo

Grupo CTO
Editorial
NOTA

La medicina es una ciencia sometida a un cambio constante. A medida que la investigacin y la experiencia
clnica amplan nuestros conocimientos, son necesarios cambios en los tratamientos y la farmacoterapia.
Los editores de esta obra han contrastado sus resultados con fuentes consideradas de confianza,
en un esfuerzo por proporcionar informacin completa y general, de acuerdo con los criterios aceptados
en el momento de la publicacin. Sin embargo, debido a la posibilidad de que existan errores humanos
o se produzcan cambios en las ciencias mdicas, ni los editores ni cualquier otra fuente implicada
en la preparacin o la publicacin de esta obra garantizan que la informacin contenida en la misma sea
exacta y completa en todos los aspectos, ni son responsables de los errores u omisiones ni de los resultados
derivados del empleo de dicha informacin. Por ello, se recomienda a los lectores que contrasten dicha
informacin con otras fuentes. Por ejemplo, y en particular, se aconseja revisar el prospecto informativo
que acompaa a cada medicamento que deseen administrar, para asegurarse de que la informacin
contenida en este libro es correcta y de que no se han producido modificaciones en la dosis recomendada
o en las contraindicaciones para la administracin. Esta recomendacin resulta de particular importancia
en relacin con frmacos nuevos o de uso poco frecuente. Los lectores tambin deben consultar
a su propio laboratorio para conocer los valores normales.

_____J

No est permitida la reproduccin total o parcial de este libro, su tratamiento informtico, la transmisin
de ningn otro formato o por cualquier medio, ya sea electrnico, mecnico, por fotocopia, por registro
y otros medios, sin el permiso previo de los titulares del copyright.

CTO EDITORIAL, S.L. 2014

Diseo y maquetacin: CTO Editorial

C/ Francisco Silvela, 106; 28002 Madrid


Tfno.: (0034) 91 782 43 30 - Fax: (0034) 91 782 43 43
E-mail: ctoeditorial@ctomedicina.com
Pgina Web: www.grupocto.es

ISBN Urologa: 978-84-16153-21-3


ISBN Obra completa: 978-84-16153-00-8
Depsito legal: M-14652-2014
Urologa

Manual CTO
de Medicina y Ciruga

9. edicin

Grupo (TO
Editorial
Q)

- t)

-e
e
04. Tumores renales. ..................... 16

_
4.1. Carcinoma de clulas renales

,_
(adenocarcinoma renal, hipernefroma) ......... _ 16

.. C)
4.2. Otros tumores . . . ......... 18

o
o
"- 05. Hiperplasia benigna de prstata
y carcinoma prosttico 20

5.1. Hiperplasia prosttica benigna... 20

01. Semiologa urolgica 5.2. Carcinoma prosttico. . ........... ...... . 22

y definiciones... ........ ................................... 1


1.1. Definiciones . . .
06. Carcinomas del tracto urinario........................ 28
1.2. Diagnstico diferencial
de la hematuria macroscpica . . . .. .... 2 6.1. Carcinoma urotelial 28
6.2. Carcinoma vesical . . . . 28
6.3. Tumores del tracto urinario superior 30

02. Infecciones del tracto urinario.


Cistitis intersticial................... ..... 3
07. Tumores testiculares 33
2.1. Patognesis y etiologa .. . .. ... ... . . ......... ...... .... ........ _. . . 3
2.2. Diagnstico... . . .... 4 7.1. Etiologa y epidemiologa . .. 33
2.3. Diferentes ITU y su tratamiento... 4 7.2. Anatoma patolgica.... . .. __ 33
2.4. Tuberculosis genitourinaria . . 6 7.3. Clnica.... ....-............ . . ... . ..... ...... .. ....... ""''"'"'"'"'"'"'' 34
2.5. Cistitis intersticial. ... . 7 7.4. Diagnstico...... . . .. . .. . . . . .. .... ... . ....... ... 34
7.5. Diagnstico diferencial 35
7.6. Tratamiento............ ..... . 35

03. Urolitiasis........ ............. 9

3.1. Epidemiologa.. .. . . . .. ........ .. 9


08. Trasplante renal. . . .......... 38
3.2. Manifestaciones clnicas
y su manejo agudo. ......... .. ....... 9 8.1. Indicaciones..... 38
3.3. Evaluacin y tratamiento 8.2. Contraindicaciones ...... .. ...... . ..... 38
de la litiasis renal... 11 8.3. Complicaciones .... ... . 38
Urologa 1 1 n d iCe

09. Uropata obstructiva ................ 40

9.1. Caractersticas. .... .. ............................... ...... 40


9.2. Clnica. . .... . . . ... ... . 40
9.3. Diagnstico. . . .... . ... ...... ........ 40
9.4. Tratamiento.......... . . . 41

1 O. Disfuncin erctil. 42
10.1. 1ntroduccin. . . ... ...... .. . . . . . . . .... . . ... . ...... ............ ... 42
10.2. Prevalencia...... . .......... . 42
10.3. Etiologa... . .. 42
10.4. Factores de riesgo . ............... ..... 42
10.5. Diagnstico..... . . ... 42
10.6. Tratamiento ..... . - 42

Bibliografa .......... ... ...... ............................. 44

----11-
Urologa

, ,
SEMIOLOGIA UROLOGICA Y DEFINICIONES

Este tema no se ha preguntado en el MIR rrgicos previos); y la segunda, el urter ectpico (que es la causa
de forma directa. Puede ayudar a obtener ms frecuente en nias).
una visin general de la materia y a asociar De esfuerzo. Se desencadena con el aumento de presin abdo
algunos hallazgos a patologas concretas,
pero no se debe emplear en l demasiado
minal (al rer, toser, cargar con peso). Generalmente se produce por
tiempo. un dficit de soporte de la musculatura perineal (por ello es reco
mendable revisar los antecedentes obsttricos, pacientes obesas,
pacientes aosas ...).
De urgencia. El paciente siente ganas de orinar, pero no le da tiem
1.1. Definiciones po a llegar al bao (provocada por contracciones involuntarias del
msculo detrusor) (MIR 09-1O, 100).
Mixta. Generalmente es una combinacin de las dos anteriores.
Hematuria microscpica. Presencia de ms de 5 hemates por campo. Paradjica. Escape de orina debido a la sobredistensin vesical. El
La causa ms frecuente en ambos sexos es la litiasis.La causa ms comn ejemplo caracterstico es el paciente prosttico con retencin uri
en varones mayores de 50 aos es la hiperplasia benigna de prstata. naria. La presin intravesical supera la presin de cierre del esfnter
Hematuria macroscpica. Orina de aspecto rojizo a simple vista debi uretral, producindose un escape de orina paradjico (no puede
do a la presencia de ms de 50 hemates por campo. En los pacientes orinar y, sin embargo, se le escapa la orina).
fumadores, en ausencia de otros sntomas, se debe sospechar tumor
urotelial.
Piuria. Presencia de ms de 1 O leucocitos por campo. Altamente ines RECUERDA
pecfica, pero en presencia de sntomas urinarios, hay que sospechar
Los cilindros hemticos aparecen en las glomerulonefritis que pro
infeccin. ducen sndrome nefrtico, como en la postinfecciosa.
Sndrome miccional. Presencia de polaquiuria (aumento en la frecuen
cia miccional), urgencia miccional (necesidad imperiosa e irrefrenable
de orinar) y disuria (molestias urinarias inespecficas referidas como ar Enuresis. Prdidas de orina exclusivamente durante el sueo. Si el nio
dor, escozor...). es mayor de 6 aos, debe ser estudiado.
Incontinencia urinaria. Prdidas involuntarias de orina (Tabla 1 ). Exis Crisis renoureteral. Dolor lumbar frecuentemente irradiado a genita
ten varios tipos principales: les, de carcter agudo, cuya intensidad no se modifica por los cambios
Continua. De da y de noche, en todas las posiciones. La causa ms posturales, y que se suele acompaar de nuseas, vmitos y malestar
frecuente es la fstula urinaria (en pacientes con antecedentes qui- general. Es muy poco frecuente que sea bilateral.

Sntomas Incontinencia de urgencia Incontinencia de esfuerzo


Urgencia (deseo repentino de orinar) S No
Aumento de la frecuencia miccional S No
Capacidad de llegar al bao despus de sentir el deseo de orinar No S
Despertarse para ir al bao durante la noche S Generalmente no
Escape durante la actividad fsica No S
Cantidad de orina escapada en cada episodio de incontinencia Abundante, si se produce Generalmente escasa
Tabla 1. Diagnstico diferencial de la clnica de incontinencia urinaria femenina

-11-
Manual CTO de Medicina y Ciruga, 9. edicin

1.2. Diagnstico diferencial Hematuria con cogulos. Indica un problema urolgico. La causa ms fre
cuente en mujeres es la cistitis hemorrgica, aunque la primera causa que se
de la hematuria macroscpica ha de descartar es una neoplasia urotelial, mxime en el paciente fumador.
Hematuria por nefropata mdica. No suele tener cogulos, y pue
Segn el momento de aparicin: de ir acompaada de cierto grado de proteinuria, as como de cilin
Inicial. Sangrado uretral o prosttico. dros eritrocitarios o de hemates dismrficos en el sedimento urinario.
Final. Sangrado del cuello vesical.
Total. Vesical o del tracto urinario alto. Se debe recordar que un
sangrado importante de cualquier parte del aparato genitourinario PREGUNTAS v' MIR09-10, 100
puede provocar hematuria total. MIR
" La hematuria con cogulos indica un problema urolgico.
Ideas clave
" La causa ms habitual de hematuria es la cistitis hemorrgica, pero
" La causa ms frecuente de hematuria microscpica es la litiasis (en lo primero que se ha de descartar es el tumor urotelial.
la poblacin general, en ambos sexos).
" Los hemates dismrficos en el sedimento orientan a nefropata de
" La causa ms comn de hematuria microscpica en varones de ms origen glomerular.
de 50 aos es la hiperplasia benigna de prstata.

01 Semiologa urolgica y definiciones


-11--
Urologa

INFECCIONES DEL TRACTO URINARIO.


CISTITIS INTERSTICIAL

Este tema es el ms importante de esta asignatura. Se debe conocer muy bien, pues dos o tres preguntas
son habituales en el examen. El estudio de las preguntas de otros aos suele ser de gran ayuda, ya que son
bastante repetitivas. No se debe bajar la guardia con la tuberculosis genitourinaria ni con la cistitis intersticial.
Hay que formar una imagen mental tpica para reconocerlas en caso clnico, que es como suelen preguntarlas.

Las infecciones del tracto urinario (ITU) pueden clasificarse de varias formas. que es del lo/o en los pacientes ambulantes, y en 3-4 das alcanza a casi la
Se puede hacer una divisin anatmica entre las ITU altas (infecciones renales) y totalidad de los pacientes sondados con sistemas de drenaje abiertos. En los
las ITU bajas (cistouretritis, prostatitis). Asimismo, la clasificacin puede basarse pacientes hospitalizados, el riesgo de infeccin alcanza un 5% por cada da
en la asociacin o no de complicaciones. Una ITU no complicada es un cua de sondaje, incluso con sistemas cerrados.
dro clnico caracterizado por la presencia de escozor miccional, urgencia y fre
cuencia, acompaado o no por hematuria terminal, dolor hipogstrico y, ms Una vez que las bacterias han alcanzado el tracto urinario, tres factores de
raramente, febrcula. Dentro de este grupo se podran incluir las pielonefritis no terminan el desarrollo de la infeccin: virulencia del microorganismo, tama
complicadas, que se presentan como cuadros febriles con hipersensibilidad en o del inculo y mecanismos de defensa del husped.
fosa lumbar, fiebre, nuseas o vmitos, y sin los factores que convierten la ITU
en "complicada': como son: presencia de catteres, uropata obstructiva, reflu La mayora de las infecciones en la comunidad estn producidas por gr
jo vesicoureteral, anomalas anatmicas, insuficiencia renal o trasplante renal. menes gramnegativos, principalmente Escherichia co/i (MIR 11-12, 203;
La ITU en el varn debe considerarse esencialmente "complicada" de entrada MIR 05-06, 230), responsable del 85% y, en menor proporcin, Proteus,
(MIR 13-14, 121). Klebsiella y Pseudomonas. Entre los grampositivos, nicamente Staphylo
coccus saprophyticus tiene relevancia, produciendo el 1O-15% de las ITU
La reaparicin de una infeccin tras el tratamiento puede deberse a reinfec en mujeres jvenes (segundo germen ms frecuente en esta poblacin).
cin o recidiva. El primer trmino expresa la infeccin nueva por un germen
distinto al inicial, mientras que recidiva indica infeccin por el mismo ger Alrededor del 30% de las mujeres con clnica miccional presentan recuen
men.Esta ltima es mucho ms infrecuente que la reinfeccin y puede estar tos menores de 1 05 unidades formadoras de colonias por mililitro ( 1 05 UFC/
ocasionada por litiasis infectiva, prostatitis crnica, fstulas vaginales o intes mi); de stas, tres cuartas partes presentan piuria; en el resto, existen pocos
tinales, divertculos vesicales infectados, cuerpos extraos, necrosis papilar datos que demuestren infeccin, y en general se tratan segn la clnica. En
infectada y otras causas que generan un reservorio de microorganismos que la orina de las pacientes sintomticas con piuria se pueden encontrar (con
difcilmente se eliminan con el antibitico. siderndose infeccin activa) recuentos ms bajos ( 1 02- 1 04) de los patge
nos habituales. En otras ocasiones, el cuadro se justifica por la presencia de
uretritis causada por Neisseria gonorrhoeae o Chlamydia trachomatis. El papel
2.1. Patognesis y etiologa patgeno de grmenes como Ureaplasma urealyticum o Mycop/asma homi
nis est mal definido, ya que se desconoce su potencial como uropatgenos
aislados.
Existen tres posibles vas por las que los microorganismos pueden alcanzar
el tracto urinario: hematgena, linftica y ascendente. La va linftica carece En las infecciones nosocomiales, los grmenes gramnegativos continan
de importancia real. La diseminacin hematgena tampoco es frecuente. La siendo los ms frecuentes. Si bien E. coli es el ms habitual, su frecuencia
ms comn es la ascendente iniciada en la uretra. Probablemente por esta desciende hasta el 50% y adquieren mayor importancia Proteus, Klebsiella,
razn es mucho ms habitual la !TU en mujeres, dado que su uretra es muy Pseudomonas, Enterobacter y Serratia. El 25% restante est ocasionado por
corta y ancha, y por ello favorece el paso de microorganismos hacia niveles grmenes grampositivos como estreptococos y estafilococos. Candida a/bi
ms altos del tracto genitourinario. Otro dato que apoya la importancia de cans puede aparecer principalmente en pacientes diabticos, cateterizados
la va ascendente es la frecuencia de infeccin tras el cateterismo uretral, o con tratamientos antibiticos prolongados.
Manual CTO de Medicina y Ciruga, 9. edicin

La afectacin del tracto urinario superior parece tambin producirse por as de tratamiento la bacteriuria persistente a los 3-5 das de haber retirado una
censo de los grmenes a lo largo del urter. La diferenciacin, aunque poco sonda vesical. En aquellos pacientes en los que la sonda no pueda ser reti
especfica, se debe basar en los hallazgos clnicos (fiebre, dolor lumbar, esca rada, el tratamiento de las bacteriurias asintomticas no suele ser efectivo, y
lofros) y analtica elemental (leucocitosis, velocidad de sedimentacin alta). puede dar lugar a seleccin de cepas resistentes.En estos pacientes slo se
debe iniciar tratamiento si presentan alto riesgo de desarrollar bacteriemia o
si la bacteriuria se hace sintomtica.
2.2. Diagnstico
En el resto de los casos, nicamente con la concurrencia de factores parti
culares se debe tratar la bacteriuria, y siempre sobre la base del estudio de
El diagnstico de la ITU, adems de por la clnica, se define por el cultivo de sensibilidades.
orina. Dado que es frecuente el crecimiento de bacterias que han contami
nado las muestras (MIR 09-1O, 209), se utiliza un criterio estadstico sobre la ITU baja en mujeres
base del recuento de colonias del urocultivo, considerando como significa
tivo clsicamente el crecimiento de ms de 1 os colonias por mililitro. Puede realizarse un tratamiento convencional de 7 das o bien un curso corto
en monodosis o en rgimen de 3 das. La ventaja de stos es el menor cos
En determinadas circunstancias, recuentos de colonias menores pueden ser te econmico y la menor incidencia de efectos adversos. Su desventaja es la
suficientes: recuentos de 1 03 UFC/ml en mujeres sintomticas, ms de 1 o en mayor incidencia de recurrencias tempranas, al no afectar apenas a los reser
pielonefritis clnicas o en varones, y ms de 101 en muestras de cateterismos vorios vaginal e intestinal de uropatgenos. Aun con todo, por las ventajas
limpios o cualquier recuento, si se recoge mediante puncin-aspiracin su mencionadas, la pauta preferida actualmente es el tratamiento de 1 - 3 das.
prapbica. Cifras mayores de 1 os UFC/ml pueden igualmente reflejar conta
minacin, principalmente si crecen dos o ms especies. Los antibiticos de eleccin son cotrimoxazol, fluoroquinolonas y P-lactmicos,
fosfomicina o nitrofurantona.
En el adulto, la presencia de piuria (ms de 1 O leucocitos/mm3) se relaciona
estrechamente con la ITU en presencia de sntomas; no as en el nio, en el En mujeres embarazadas se recomiendan las pautas largas de tratamiento
que puede acompaar a los cuadros febriles. (7 das), evitando el uso de sulfamidas al final del embarazo (tercer trimes
tre), por el riesgo incrementado de kernicterus, y el empleo de quinolonas
por el dao producido sobre el cartlago de crecimiento fetal.Tampoco se
2.3. Diferentes ITU y su tratamiento emplearn pautas cortas en caso de sospecha de pielonefritis, presencia
de clculos o anomalas de la va urinaria, o bien infecciones previas por
microorganismos resistentes a los antibiticos.
En el tratamiento de la ITU, lgicamente, es fundamental el uso de anti micro
bianos.El nmero empleado de stos es elevado y las pautas de tratamiento ITU recurrente
son muy variables. A continuacin, se repasarn las opciones teraputicas
segn el tipo de ITU a la que se haga frente. Aparicin de cuatro o ms episodios al ao. Se puede realizar profilaxis
con cotrimoxazol o una fluoroquinolona (en funcin de la sensibilidad
Bacteriuria asintomtica del germen aislado en el ltimo episodio) en dosis nica, das alternos,
durante 6 meses. Si tras la retirada se presentaran nuevas recurrencias,
Definida como bacteriuria significativa (105 UFC/ml) en al menos 2 uroculti puede reinstaurarse el tratamiento durante periodos ms prolongados
vos con el mismo germen, tomados con 1 semana de diferencia en ausencia (1 -2 aos). Es aconsejable la ingesta abundante de agua y realizar miccio
de sntomas. La bacteriuria asintomtica no debe tratarse salvo en los casos nes frecuentes y cumplir una serie de reglas bsicas higinico-dietticas.
en los que conlleva un riesgo de infeccin clnica o dao orgnico.Debe ser
tratada en los siguientes supuestos: Si los episodios tienen relacin con el coito, se puede administrar un compri
Embarazadas (MIR 08-09, 98; MIR 05-06, 102; MIR 03-04, 78). mido de cotrimoxazol o una quinolona despus del mismo. En mujeres pos
Pacientes inmunodeprimidos. menopusicas, el tratamiento con estrgenos tpicos vaginales disminuye
Profilaxis previa a ciruga urolgica. la frecuencia de infecciones.
Bacteriuria por Proteus.

RECUERDA
RECUERDA S. saprophyticus se ha relacionado con /TU en mujeres jvenes
Proteus es intrnsecamente resistente a las nitrofurantofnas sexualmente activas.
(MIR 03-04, 54), ya que alcaliniza la orina gracias a su ureasa, y
este grupo de antibiticos nicamente es til en medio cido.
Pielonefritis aguda no complicada
En el caso de los pacientes sondados permanentemente, la presencia de En los casos de gravedad leve-moderada, puede plantearse teraputica oral con
bacteriuria asintomtica no es una indicacin de tratamiento y, actualmente, cotrimoxazol (en desuso en nuestro medio por el elevado ndice de resistencias),
incluso es dudosa la recomendacin clsica de empleo profilctico de algn fluoroquinolonas o P-lactmicos. En pacientes graves u hospitalizados es preciso
antibitico, previo a la sustitucin del catter, a fin de contrarrestar la posible tratamiento parenteral, y el espectro de antimicrobianos incluye ampicilina (en
diseminacin hematgena del germen producida por la manipulacin (las terococo), ureidopenicilinas (Pseudomonas), cefalosporinas de segunda o tercera
ltimas guas clnicas ya no lo recomiendan). Sin embargo, s es indicacin generacin, e incluso aminoglucsidos. Nunca se emplearn pautas cortas.

02 I nfecciones del t r acto u r i n a rio. C i s titis i n tersticia l --11--- -


Urologa I O2
El antibitico empleado se seleccionar, por supuesto, sobre la base del cul RECUERDA
tivo y del antibiograma, y cuando se inicie de forma emprica, habr que
En pacientes con SIDA, Cryptococcus neoformans puede ser una
tener en cuenta factores que orienten hacia el germen causante: mayor inci causa de prostatitis, ya que se elimina a travs de la orina.
dencia de Pseudomonas en personas diabticas y pacientes de UVI, estafilo
coco en adictos a drogas parenterales (MIR 03-04, 84), Proteus en pacientes
con litiasis infectiva (MIR 09-1 O, 101 ), presencia de sondas, catteres, trata La prostatitis crnica bacteriana suele presentarse como molestias perinea
mientos antibiticos previos, etc. les o genitales, sntomas irritativos (polaquiuria, tenesmo, escozor) y epi
sodios de ITU recurrentes causados por el mismo organismo. En el lquido
En la evaluacin del paciente con pielonefritis y mala respuesta al tratamien prosttico se evidencian ms de 1 O leucocitos por campo de gran aumen
to inicial, es recomendable la realizacin de una ecografa para descartar to, y macrfagos que contienen cuerpos ovales grasos. El tratamiento debe
obstruccin o litiasis. estar guiado por los cultivos, tanto de orina como de fluido obtenido por
masaje prosttico, y prolongarse entre 4-16 semanas.
ITU en varones
Cuando se encuentra a un paciente con datos de prostatitis crnica y signos
Cualquier ITU en varn debe considerarse como complicada inicialmente, inflamatorios en el lquido prosttico, pero sin historia documentada de ITU
ya que hay que asumir que existe afectacin del tejido prosttico, renal o y con cultivos negativos, el cuadro se denomina prostatitis crnica no bac
que existen problemas concomitantes como obstruccin urinaria, litiasis o teriana. En ocasiones, el responsable puede ser U. urealyticum o M. hominis,
malformaciones urolgicas (MIR 07-08, 93). pudiendo ser tratados estos casos con doxiciclina o eritromicina, sobre esta
sospecha.
Por todo ello, el tratamiento debe ser ms prolongado (mnimo, 1 semana),
no siendo adecuados los cursos cortos de tratamiento (MIR 07-08, 125). Se denomina prostatodinia a un cuadro clnico similar donde predominan
las molestias perineales o genitales con cultivos negativos y menos de 1 0
Prostatitis leucocitos por campo en el lquido prosttico. S u causa e s desconocida y
el tratamiento difcil, emplendose actualmente a-bloqueantes o relajantes
La infeccin aguda del tejido prosttico se presenta como un cuadro sptico musculares como terapia inicial.
con afectacin general del paciente, fiebre elevada, sndrome miccional, a r
tromialgias y dificultad miccional (Tabla 2). En el examen rectal, la prstata Orquiepididimitis
aparece muy dolorosa e inflamada. El germen ms habitual es E.coli. Durante
la inflamacin aguda, los antibiticos penetran adecuadamente, pero una vez En varones adultos menores de 35 aos es considerada, en el plano terico,
que sta cede, la penetracin es ms pobre. Por ello, se deben utilizar cursos una enfermedad de transmisin sexual, siendo los agentes ms frecuentes
largos de tratamiento (3-4 semanas) para intentar evitar la persistencia de fo C. trachomatis y N. gonorrhoeae (Tabla 3). Por encima de 35 aos, los mi

--
cos que den pie a una prostatitis crnica. Entre los antimicrobianos emplea croorganismos ms frecuentes son las enterobacterias. El tratamiento pue
dos, las fluoroquinolonas son las que mejor difunden al tejido prosttico. de llevarse a cabo con las siguientes pautas: 1 ) quinolonas, 2) ceftriaxona

Prostatitis
EME
E. coli
Clnica

Cuadro sptico + +
Lquido prosttico Cultivo lquido
prosttico

Nunca hacer masaje prosttico ni sondaje


Tratamiento

Cotrimoxazol,
aguda fluoroquinolonas 4 semanas
Prostatitis crnica E. coli Irritativo con + +!- > 1 O leucocitos/campo + Cotrimoxazol,
bacteriana reagudizaciones, sin fluoroquinolonas
fiebre ni leucocitosis 4-1 6 semanas
Prostatitis crnica Ureaplasma Cronicidad. > 1 O leucocitos/campo Doxiciclina
no bacteriana Mycoptasma empeoramiento
Prostatodinia Desconocida Oscilante < 1 O leucocitos/campo u-bloqueantes
Relajantes musculares
Tabla 2. Diagnstico diferencial de las prostatitis

Enfermedad Etiologa Lesin tpica Diagnstico Tratamiento


Uretritis N. gonorrhoeae Asintomtica ( !? ) Contacto < 5 das Ceftriaxona o espectinomicina
gonoccica Exudacin uretral matutina ( ! ) Gram de exudado cervical (no en farngeas)
Epididimoprostatitis, salpingitis, Cultivo en medio Ciprofloxacino
sndrome Fitz-Hugh-Curtis, de Thayer-Martin
gonococemia diseminada (dficit C5-C8,
menstruacin, embarazo, auxotipo AHU)
Uretritis C. trachomatis Similar a las UG, pero con menos signos Contacto 7-1 5 das. Excluir Tetraciclinas o macrlidos
no gonoccica U urealyticum y sntomas gonorrea por Gram y cultivo.
Epididimitis, proctitis, cervicitis, EIP C. inclusin-Giemsa IFD,
medios celulares
Tabla 3. Diagnstico diferencial de las uretritis

11-
Manual CTO de Medicina y Ciruga, 9. edicin

en dosis nica i.m. (1 25-250 mg) ms 1 O das de doxiciclina (100 mg/12 h/7 El diagnstico es similar al absceso renal, y su tratamiento pasa por el drena
das); se aplicar esta pauta en aquellos casos en los que se sospeche enfer je percutneo o quirrgico, con la adecuada cobertura antibitica.
medad de transmisin sexual.
ITU asociada a catteres
Absceso renal
La ITU es la infeccin hospitalaria ms frecuente, y los catteres urinarios la
Los abscesos medulares o corticales suelen proceder de un foco de pielone principal fuente de sepsis. Se calcula que el l o/o de cateterismos ambulato
fritis contiguo o de diseminacin hematgena de Stophylococcus aureus, pro rios transitorios sufren una ITU posterior y que la mayora de pacientes con
cedente de focos cutneos en usuarios de drogas por va parenteral (UDVP). catter permanente presentan una bacteriuria significativa al cuarto da de
El urocultivo en este ltimo caso puede ser negativo. El diagnstico ms fiable su colocacin. Esta bacteriuria puede hacerse sintomtica en forma de cua
se realiza mediante tomografa computarizada (TC). Deben tratarse con anti dros de cistitis, hematuria o episodios febriles, muchas veces autolimitados.
biticos por va intravenosa y, dependiendo del tamao y de la evolucin, se
hace obligatorio el drenaje mediante puncin percutnea o quirrgicamente. Entre los factores que aumentan el riesgo de ITU asociada a catter urinario
se encuentran: sexo femenino, edad avanzada, mala tcnica de sondaje, sis
Absceso perirrena I temas de drenaje abiertos y falta de higiene local.

Se localiza entre la cpsula renal y la fascia de Gerota. Lo ms frecuente es Entre los antibiticos disponibles, parece que las quinolonas son los que
que un absceso cortical se abra a este espacio, pero puede ocurrir tambin mejor eliminan la pelcula biolgica de los catteres infectados, favorecien
por diseminacin hematgena. El germen ms frecuente es E. co/i, y S. au do as el tratamiento de la infeccin; en cualquier caso, ste nicamente
reus en los casos de diseminacin hematgena (Figura 1 ). se recomienda si existe sintomatologa o en el momento de la retirada del
catter, por el mayor riesgo de ITU sintomtica y sepsis.

2.4. Tuberculosis genitourinaria

Generalmente est ocasionada por Mycobacterium tuberculosis. El aparato


genitourinario es el sitio ms frecuente de afectacin extrapulmonar (tras
la adenitis tuberculosa). Un So/o de los pacientes con tuberculosis activa pre
senta afectacin del tracto genitourinario (Figura 2).

Tras la inhalacin del bacilo, se produce una diseminacin hematgena (pri


moinfeccin) con siembra de bacilos en ambos riones en el 90% de los
casos. Sin embargo, la enfermedad clnica generalmente es unilateral.

Obliteracin
ureteral

Amputacin Pionefrosis
calicial

Microvejiga

Prstata
y vesculas
seminales
Figura 1 . TC en la que se muestra un absceso perirrenal

Estenosis
ureteral distal 1 Epididimitis
/
RECUERDA 1
S. aureus es tambin la causa ms frecuente de endocarditis infec
ciosa. Figura 2. Lesiones de la tuberculosis genitourinaria

02 l o f m i o m d e l ! < a c t o " ' i oa , i o . C i s t i t i s i o t m t i e i a l --11-


f
Urologa I O2
El eriodo de latencia entre la "siembra" y la enfermedad clnica oscila en 2.5. Cistitis intersticial
tr/l 0-40 aos, afectando principalmente a pacientes por debajo de los 50
aos. La lesin inicial microscpica se localiza en los glomrulos en forma de
ranulomas microscpicos. Al avanzar la enfermedad, se produce afectacin Aunque no es un cuadro infeccioso, se incluye en el presente captulo esta
s distal hasta la aparicin de una papilitis necrotizante, momento en el entidad inflamatoria vesical de origen desconocido.
que ya puede existir paso de bacilos a la va excretora donde, por procesos
inflamatorios, ocasionar estenosis a nivel de los infundbulos caliciales, pel En este sentido, se esgrimen dos teoras no demostradas. Por un lado, la
vis y urter, con hidronefrosis secundaria. Las lesiones renales pueden cavi teora autoinmunitaria; por otro, la de un dficit en el recubrimiento urotelial
tarse y calcificarse, y llegar a producir una destruccin total del parnquima por glucosaminoglucanos.
(fenmeno que se denomina "rin mastic").
Clnica
Clnica
Suele presentarse en mujeres entre 30-70 aos, como un cuadro cisttico
Los hallazgos clnicos son escasos. En el 70% de los pacientes, los sntomas crnico en el que destacan disuria, polaquiuria con nicturia y dolor suprap
son leves. Lo ms frecuente es la aparicin de microhematuria, dolor vago bico, acompaados en ocasiones de hematuria (20-30%).
en flanco o clico renal. La afectacin vesical, sin embargo, s produce sin
tomatologa florida con un sndrome cisttico rebelde, donde la polaquiuria
(secundaria a la disminucin de la capacidad vesical) es lo ms llamativo. En RECUERDA
varones, es frecuente la aparicin de una orquiepididimitis crnica que no
Existen muchas ms causas de sndrome cisttico: cistitis aguda, tu
responde a la terapia habitual. berculosis, carcinoma in situ, entre otras.

En el 90% de los pacientes, el anlisis urinario es anormal. Tpicamente apa


rece piuria cida con urocultivo negativo (MIR 10-1 1 , 1 06). La prueba
de laboratorio ms importante es el cultivo de M. tuberculosis en medio se Diagnstico
lectivo (Lwenstein), ya que los medios de tincin rpida (Ziehl. auramina),
aunque vlidos, pueden dar falsos positivos por contaminacin con Myco El diagnstico es bsicamente por exclusin de otra patologa que pueda
bacterium smegmatis. ocasionar un cuadro similar (infeccin bacteriana, tuberculosis, litiasis o tu
mor vesical) apoyado en los hallazgos cistoscpicos sugestivos:
Diagnstico Petequias submucosas, principalmente trigonales, que aparecen al dis
tender la vejiga (glomerulaciones).
El cultivo en medio de Lwenstein es positivo en el 90% de los pacientes lceras de Hunner.
con enfermedad activa, aunque deben obtenerse, al menos, 3 muestras de
das diferentes para mejorar la sensibilidad, ya que el paso de bacilos a orina La biopsia vesical. adems de descartar la presencia de carcinoma in situ u
no es constante.Actualmente, lo ms rentable es realizar una PCR de orina otra patologa, revela en algunos casos, un infiltrado intersticial de mastocitos.
en busca del ARN del bacilo.
Tratamiento
Radiolgicamente, el 90% de los pacientes presentan urogramas alterados.
El hallazgo ms sugestivo es la presencia de cavidades que comunican con Aunque esta enfermedad raramente supone una amenaza para la vida de la
el sistema colector. Inicialmente estas cavidades son mnimas y dan un as paciente, su morbilidad es elevada.
pecto "mordisqueado" a los clices. Segn la enfermedad avanza, pueden
encontrarse estenosis infundibulares, ureteropilicas, en unin ureterovesi Desgraciadamente, las diversas alternativas de tratamiento nicamente
cal o vejigas pequeas de aspecto rgido. En el punto ms evolucionado de pueden encaminarse a una mejora sintomtica, en la mayora de los casos
la enfermedad, el rin puede encontrarse anulado, disminuido de tamao con resultados discretos:
y con calcificaciones parenquimatosas. Distensin hidrulica vesical.
Amitriptilina oral.
Tratamiento Instilacin con dimetilsulfxido (DMSO).
Corticoides tpicos o sistmicos.
El tratamiento mdico de la enfermedad activa no difiere sustancialmente Denervacin vesical.
del de la tuberculosis pulmonar en cuanto a frmacos y periodo de trata Cistoplastias de aumento.
miento. Puede ser necesario el tratamiento quirrgico, dependiendo de la Cistectoma.
complicacin asociada, generalmente estenosis de la va excretora e hidro
nefrosis. En caso de rin no funcionante por lesin extensa del parnqui
ma, puede ser precisa la nefrectoma. v' MIR 13-14, 121
v' MIR 11-12, 203
v' MIR 10-11, 106
v' MIR 09-1O, 101, 209
RECUERDA v' MIR 08-09, 98
v' MIR 07-08, 93, 125
Rifampicina, isoniacida y pirazinamida son los tres antibiticos ms
v' MIR 05-06, 102, 230
empleados en el tratamiento de la tuberculosis.
v' MIR 03-04, 54, 78, 84

11
Manual CTO de Medicina y Ciruga, 9. edicin

" El diagnstico definitivo de ITU es microbiolgico: ms de 1 os


Ideas c lave UFC/ml. No obstante, este criterio vara con el sistema de recogida.

" La causa ms frecuente de ITU es E. coli, tanto a nivel comunitario " Si se recoge la muestra urinaria mediante puncin suprapbica,
como nosocomial. cualquier nmero de bacterias es significativo.

" El origen ms frecuente de uretritis es C. trachomatis. " La bacteriuria asintomtica se trata en gestantes, inmunodeprimi
dos, previamente a la ciruga urolgica, o si la especie implicada es
" La causa habitual de orquiepididimitis depende de la edad: Ch/a Proteus.
mydia y gonococo si es menor de 35 aos; enterobacterias, si es
mayor de esa edad. " Los sistemas de drenaje cerrados son preferibles a los abiertos, pues
la tasa de infeccin es menor.
" La causa ms frecuente de absceso renal en UDVP es S. aureus.

4) TBC urogenital.
C asos clnicos ., , 5) Pielonefritis crnica por Pseudomonas aeruginosa.

Un prosttico, sin otros problemas de salud, portador de sonda ure RC: 4


tral permanente, presenta bacteriuria (> 1 os unidades formadoras
de colonias) en dos urocultivos. Cul es la actitud teraputica ms Una paciente de 27 aos acude al servicio de Urgencias por dolor
conveniente? en fosa renal derecha, fiebre de 39 C, escalofros y sndrome mic
cional acompaante. Es alrgica a penicilinas. Seale la respuesta
1) Tratamiento antibitico de amplio espectro. correcta:
2) Tratamiento antibitico segn antibiograma.
3) Cambio de sonda urinaria exclusivamente. 1 ) No ser necesario descartar patologa urinaria obstructiva en este
4) Antispticos en vejiga urinaria. caso, ya que presenta un claro sndrome miccional.
5) Cambio de sonda urinaria y tratamiento antibitico. 2) Para poder hacer el diagnstico de pielonefritis se deber conocer
primero los datos referidos a la funcin renal.
RC: 3 3) Se deber iniciar tratamiento emprico con un -lactmico.
4) Si en las primeras horas evoluciona favorablemente, podr conti
Ante un paciente de 24 aos que presenta fiebre alta con dolor, in nuar el tratamiento de forma ambulante.
flamacin y enrojecimiento testicular izquierdo, cul de las siguien 5) El mejor tratamiento disponible es la administracin intramuscular
tes afirmaciones es INCORRECTA? de aminoglucsidos.

1 ) El diagnstico ms probable es epididimitis. RC:4


2) Los patgenos ms frecuentes son Chlamydia trachomatis y Neisse-
ria gonorrhoeae. Un paciente de 83 aos sondado de forma permanente acude a la
3) El tratamiento de eleccin es vancomicina + gentamicina. consulta tras detectrsele 2 cultivos positivos tomados con 1 sema
4) El tratamiento de eleccin puede ser ofloxacino. na de diferencia. Asegura encontrarse asintomtico. La actitud ms
5) Un tratamiento alternativo es ceftriaxona en monodosis ms 1 O adecuada ser:
das de doxiciclina.
1 ) Iniciar tratamiento antibitico segn antibiograma de los cultivos
RC: 3 obtenidos.
2) Tranquilizar al paciente y seguir con su pauta habitual de recambio
Ante un paciente que presenta febrcula persistente, crisis renou de sonda.
reterales breves, piuria estril, orina con pH cido, microhematuria 3) Realizar cambio de sonda de forma inmediata con tratamiento an
persistente, con citologa urinaria negativa y epiddimos indurados, tibitico.
en qu enfermedad se debe pensar primero? 4) Realizar cambio de sonda de forma inmediata con profilaxis anti
bitica de 4 das.
1) Sarcoidosis. 5) Retirar la sonda y colocar cistotoma suprapbica.
2) Carcinoma vesical.
3) Carcinoma renal. RC: 2

0 2 I n f e c c i o n e s del tra cto u r i n a r i o . C i s t i t i s i n t e r s t i c i a l


11
p

Urologa

UROLITIASIS

ORI ENTAC I N Tema fundamental en esta asignatura. Se debe conocer perfectamente la actitud ante la litiasis en general,

MIR ante los distintos tipos de clculos y, especialmente, todo lo relacionado con el tratamiento. Es un tema rentable
y agradecido, as que hay que emplear el tiempo necesario. La tabla-resumen de urolitiasis puede ser de gran ayuda.

3.1 . Epidemiologa 3.2. Manifestaciones clnicas


y su manejo agudo
Son numerosas las sustancias que se han identificado formando parte de los
clculos. Su incidencia vara segn el pas, e incluso segn las reas geogr El dolor agudo del clico renal es la manifestacin ms tpica de la litiasis
ficas dentro del mismo pas. renal. El dolor se produce por la sobredistensin de la va urinaria tras la obs
truccin de sta por el clculo. Es lgico, por tanto, que el clculo deba des
Se pueden distinguir seis grupos de componentes: plazarse desde su origen calicial para producir sintomatologa aguda. Oca
Oxalato clcico. sionalmente se observan cuadros de dolor vago renal en relacin con litiasis
Fosfato clcico. caliciales no desplazadas. El clico renal o crisis renoureteral suele aparecer
Fosfato no clcico. de forma progresiva sobre la fosa lumbar afectada, irradindose por el flanco
Compuestos purnicos (cido rico; urato amnico; urato sdico; xanti hacia la ingle y los genitales (Figura 3). El paciente generalmente se en-
na; 2,8 dihidroxiadenina).
Aminocidos (cistina).
Otros (carbonato clcico, sulfamidas, etc.).

Los clculos de oxalato clcico son los ms frecuentes, con cifras en torno
al 65%, seguidos por los infectivos y de cido rico (alrededor del 1 5% cada
uno), los de fosfato clcico (5%) y los de cistina (1 -3%). La tercera dcada
es la edad media de aparicin, por primera vez, de la litiasis; salvo en los de
cistina, que suelen ser de aparicin ms prematura.
RX simple de abdomen
En Espaa, la incidencia de la litiasis alcanza al 4,2% de la poblacin, con ma con clculo a nivel de L3

yor afectacin de varones que mujeres. nicamente los clculos infectivos


tienen mayor incidencia en la mujer.

RE C U ERDA
Las infecciones urinarias son ms frecuentes en mujeres que en va
rones. Por eso los clculos de estruvita tambin lo son. Dolor irradiado Sndrome miccional
Hematuria

La enfermedad litisica recidiva en el 40% de los casos, con una media de


un nuevo clculo cada 2-3 aos. Por recidiva se entiende la aparicin de una
nueva litiasis de la misma composicin y en la misma localizacin, en un
intervalo menor de 4 aos entre un clculo y otro. Figura 3. Diagnstico de urolitiasis

11
Manual CTO de Medicina y Ciruga, 9. edicin

cuentra afectado, con dolor que no cede con reposo, por lo que cambia de yora de los clculos son radioopacos, exceptuando los de cido rico y
postura continuamente. Puede acompaarse de un cortejo vegetativo con algunas otras composiciones infrecuentes (sulfamidas, xantina, indinavir)
nuseas, vmitos y sudoracin. El dolor irradiado hacia la ingle suele indicar (MIR 06-07, 93; MIR 04-05, 1 04).
que el clculo ha alcanzado el urter.Cuando se encuentra en vecindad de
la vejiga, o bien dentro de sta, puede aparecer un cuadro irritativo, similar al El estudio de imagen se completar mediante otras tcnicas diagnsticas.
sndrome miccional con polaquiuria, disuria y tenesmo vesical. La ecografa permitir visualizar incluso las litiasis radiotransparentes, con
el inconveniente de no ser vistas aqullas ubicadas en el trayecto ureteral
Los clculos infectivos de estruvita, y en menor medida los de cido rico (salvo las zonas cercanas a la vejiga o al rin).Tambin se podr evaluar el
y cistina, pueden crecer modelando las cavidades renales (litiasis coralifor grado de hidronefrosis (MIR 13-14, 21 ).
me o "en asta de venado" [Figura 4]), manifestndose no como clico, sino
como infecciones urinarias de repeticin, dolor lumbar sordo, hematuria o La urografa ofrece informacin morfolgica y funcional de ambos riones
incluso insuficiencia renal. (Figura 5). Debe tenerse en cuenta que, durante el clico renal, puede ob
servarse una anulacin funcional, sin que signifique necesariamente dete
rioro de dicha unidad renal. Mediante esta tcnica se puede diagnosticar
todo tipo de clculos, ya sean radiotransparentes o radioopacos. El principal
inconveniente de este procedimiento es la introduccin de contraste yoda
do, que est contraindicado en los pacientes con alergia, creatinina mayor
de 2, mieloma mltiple o deshidratacin importante.

Segn las guas clnicas, la urografa intravenosa (UIV) actualmente ha sido


desplazada por la TC helicoidal sin contraste, que se ha convertido en el
nuevo estudio de referencia para las litiasis. Aunque su alto coste hace que
todava no est extendido su uso, permite evaluar todo tipo de clculos.

Figura 4. Litiasis coraliforme o "en asta de venado"

Diagnstico
El anlisis bsico de orina muestra generalmente hematuria y leucocituria.
Una piuria importante apoyara la posibilidad de infeccin sobreaadida,
aunque ninguno de estos datos es realmente determinante.

Los cristales de oxalato clcico dihidratado aparecen como bipirmides tetra


gonales al observarlos con lupa binocular. Los de oxalato clcico monohidra
tado aparecen como cristales alargados que adoptan forma de empalizada, Figura 5. (A) Litiasis ureteral; (B) UIV de uropata obstructiva izquierda
formando clculos de estructura radiada, con aspecto compacto y macizo.
Tratamiento
Entre los fosfatos clcicos, la brushita es el compuesto ms cido, formando
cristales grandes en forma de abanico de color azul con luz polarizada. Las El manejo agudo del clico renal se basa en el control del dolor. Para esto,
apatitas tienen aspecto microgranular o esferoctico. El cido rico aparece es preciso conseguir una disminucin de la presin dentro de la va urinaria,
bajo la lupa como una desordenada aglomeracin de cristales. En algunos lo que puede hacerse, sobre todo, con antiinflamatorios, que disminuyen
clculos, los cristales estn tan juntos que se asemejan a una masa continua. el dolor y la diuresis al inhibir la sntesis de prostaglandinas. En ocasiones,
stos resultan insuficientes o estn contraindicados (por ejemplo, en la insu
La estruvita (o fosfato amnico magnsico) es el componente ms carac ficiencia renal), pudindose usar tambin otros frmacos como los opiceos.
terstico de los clculos producidos por infeccin por grmenes urealticos. Asimismo, se pueden usar espasmolticos, que disminuyen la presin intrau
Sus cristales tienen formas prismticas polimorfas, y raramente se observan reteral al relajar la pared del urter.
los cristales "en atad" que pueden hallarse en el sedimento. La cistina se
reconoce fcilmente por su aspecto acaramelado, formando cristales hexa Existe una serie de situaciones en las que el clico renal se convierte en una
gonales en prismas o lminas. urgencia que precisa de hospitalizacin y, eventualmente, de manipulacin
invasiva (derivacin urinaria):
En teora, el 90% de los clculos son visibles en una radiografa simple de Obstruccin grave, principalmente si se acompaa de litiasis mayor de
abdomen, aunque este porcentaje es considerablemente menor en las l O mm.
radiografas urgentes sin preparacin intestinal. Radiolgicamente, la ma- Fiebre elevada (> 38 C) (MIR 1 3-14, 22).

03 U r o l i t i a s i s
-11-
..
Urologa I O3
Dolor incontrolable. Litiasis clcica Litiasis Litiasis infectiva
(oxalato o fosfato) rica (estruvita)
Rin nico.
Hipercalciuria Gota primaria Cistinuria Infecciones
ASimismo, en pacientes diabticos, por el mayor riesgo de complicaciones, idioptica Hemopatas por grmenes
es aconsejable, si no el ingreso, al menos una observacin estricta. Una si Hipercalciuria Enfermedades productores
tuacin similar ocurre durante el embarazo, donde una dilatacin leve de la secundaria digestivas de ureasa
va urinaria puede considerarse "fisiolgica: pero obstrucciones ms impor a hipercalcemia lngesta
tantes o la aparicin de fiebre hacen aconsejable la colocacin de un catter Hiperuricosuria excesiva
ureteral (Figura 6). Hiperoxaluria de purinas
Hipocitraturia Frmacos
Acidosis tubular Litiasis rica
renal distal idioptica
Litiasis clcica
idioptica
Tabla 4. Tipos de litiasis. Situaciones que favorecen su aparicin

RECUERDA
La furosemida, al revs que las tiacidas, aumenta el calcio urinario.

Este estudio debe reservarse para aquellos pacientes con alta probabilidad
de recidiva, aunque cada vez ms autores indican que debe realizarse a to
dos los pacientes. Son indicaciones de estudio metablico las siguientes:
Edad temprana de aparicin.
Litiasis bilateral.
Litiasis en rin nico o malformado.
Composicin poco frecuente.
Litiasis recidivante.
Nefrocalcinosis.
Litiasis coraliforme.

Desde el punto de vista prctico, las litiasis se pueden dividir en las de com
posicin clcica y las de otras composiciones, ya que el primer grupo supo
ne la mayora de los casos (70-80%) tratados habitualmente.

Litiasis clcica

En la mayora de las ocasiones se desconoce el origen de la litiasis clcica,


aunque se puede hacer una aproximacin a los factores de riesgo que in
fluyen en su aparicin. Slo en un pequeo porcentaje de casos existe una
enfermedad de base que puede ser tratada, y de esta forma desaparece la
formacin de clculos clcicos.
Figura 6. Doble J derecho. Litiasis ureteral derecha. Litiasis coraliforme Hipercalciuria idioptica. Es la causa ms frecuente de litiasis clcica.
izquierda Se define como una excrecin urinaria de calcio mayor de 300 mg/24
h en el varn y 250 mg/24 h en la mujer. De cara a su manejo, las tia
zidas disminuyen el calcio urinario, reduciendo la formacin de litiasis
3.3. Evaluacin y tratamiento (MIR 03-04, 44). La administracin de citrato potsico ayuda a evitar
la hipopotasemia y aumenta el citrato urinario, que es inhibidor de la
de la litiasis renal litognesis (Tabla 5).

Este apartado se puede dividir en dos partes. Por un lado, el estudio de la


litiasis con la finalidad de instaurar un tratamiento preventivo de su forma Absortivas Resortivas Renales
cin; y por otro, el estudio y tratamiento de la litiasis ya formada. Aporte excesivo Hiperparatiroidismo Acidosis tubular
Sndrome de Burnett Inmovilizacin distal
Estudio y tratamiento preventivo (leche y alcalinos) Tumorales ldioptica
Hipervitaminosis D Enfermedad de Paget
La evaluacin del paciente con litiasis se basa en un estudio metablico para ldioptica Sndrome de Cushing
determinar qu factores son modificables, en un intento de evitar la recidiva Sarcoidosis
(Tabla4). Tabla S. Causas ms frecuentes de hipercalciuria
Manual CTO de Medicina y Ciruga, 9. edicin

Hiperuricosuria. Excrecin en orina de ms de 800 mg/24 h en el varn o son los que mejor responden al tratamiento mdico mediante quimilisis por
750 mg/24 h en la mujer. Adems de favorecer la litiasis rica, la hiperurico alcalinizacin urinaria.Pueden administrarse diversos lcalis; el citrato potsi
suria constituye un factor de riesgo para la formacin de clculos de calcio, co impedira el terico riesgo de formacin de clculos clcicos por su efec
probablemente por nucleacin heterognea sobre ncleos de cido rico to inhibidor, pero tambin pueden tratarse con bicarbonato sdico o citrato
o urato sdico.Generalmente se debe a un exceso de purinas en la dieta. sdico. Una alternativa es la acetazolamida en dosis de 250 mg/da. Cuando,
Hiperoxaluria. Se considera como tal la excrecin en orina de ms de adems, la uricemia es alta, puede tratarse con alopurinol.
40 mg/24 h. Existe una hiperoxaluria primaria, que es consecuencia de
un defecto enzimtico autosmico recesivo; no tiene tratamiento y ge L i t i a s i s cistn i ca
neralmente conduce a insuficiencia renal por litiasis recidivante. El ni
co tratamiento que existe actualmente es el trasplante heptico, que La cistinuria es un trastorno autosmico recesivo en el que existe un defecto
suele ir unido al renal, aunque algunos casos responden a piridoxina. de absorcin, a nivel intestinal y tubular proximal, de los aminocidos dib
sicos: cistina, ornitina, lisina y arginina (COLA), aunque parece que puede
existir un trastorno en el que nicamente se ve afectada la cistina, lo que
RE C UER D A indicara que, adems de un mecanismo de transporte comn, existe uno
independiente para la cistina.
La causa ms frecuente de hipercalcemia en un paciente ambulato
rio es el hiperparatiroidismo primario. En cambio, la hipercalcemia
ms frecuente en uno ingresado es la de origen neoplsico. Los niveles de cistina en orina de 24 horas son superiores a 100 mg; de he
cho, los homocigotos pueden excretar ms de 600 mg/da. El diagnstico
se realiza identificando los caractersticos cristales hexagonales en orina, o
No obstante, la mayora de los casos de hiperoxaluria son secundarios a por una prueba positiva de nitroprusiato sdico (la orina se tie de azul en
malabsorcin de cidos grasos por enfermedades crnicas pancreatobilia pacientes afectados por esta enfermedad: test de Brand).
res, derivacin intestinal para el tratamiento de la obesidad mrbida. resec
cin ileal, enfermedad inflamatoria intestinal (MIR 05-06, 104), hipercal El tratamiento consiste en aumentar la diuresis diaria (ms de 3 1/da), al
ciuria coincidente o por falta de calcio en la dieta, lo que permite que exista calinizar la orina por encima de 7,5 y, en caso de que esto sea insufi
mayor cantidad de oxalato intestinal para su absorcin.La intoxicacin por ciente, puede iniciarse tratamiento con Dpenicilamina (250 mg/6 h) o
etilenglicol y metoxiflurano puede producir hiperoxaluria, as como la in a.mercaptopropionilglicina (250 mg/6 h).
gesta de vitamina C en altas dosis. En todos estos casos secundarios. el
tratamiento con colestiramina, una dieta pobre en grasas y la correccin L i t i a s i s i nfectiva
de la malabsorcin, en la medida de lo posible, suelen ser medidas eficaces.
Hipocitraturia. Excrecin de citrato inferior a 300 mg/24 h. General Los clculos infectivos de estruvita o de fosfato amnico magnes1co
mente se asocia a otras anomalas urinarias. Aunque de causa descono (MgNH/04-6Hp) se desarrollan en un ambiente alcalino, producido por
cida, puede contribuir una dieta rica en protenas, hipocaliemia, enfer infeccin persistente de grmenes que hidrolizan la urea, aumentando la
medad intestinal o infeccin urinaria. cantidad de amonio urinario. Los principales grmenes que poseen ureasa,
Hiperparatiroidismo primario. Supone la causa ms frecuente de hipercal adems de diversas especies de Proteus (MIR 1 0- 1 1 , 116; MIR 06-07, 1 06),
ciuria conocida (vase Seccin de Endocrinologa, metabolismo y nutricin). son Pseudomohas, Klebsiella, Serratia y Enterobacter. La presencia de cuerpos
Acidosis tubular renal distal (vase Seccin de Nefrologa). Enfermedad extraos (sondas vesicales, suturas) favorece su formacin.
autosmica recesiva. Consiste en la imposibilidad del tbulo distal para
excretar hidrogeniones a la orina (orinas persistentemente alcalinas) con Para su tratamiento se han empleado diversos mtodos, generalmente inefi
aumento de la eliminacin de calcio a la orina.Existen formas incomple caces.La antibioterapia nicamente mantiene estril la orina durante los cur
tas que se observan en pacientes formadores de clculos de oxalato cl sos de tratamiento. Parece ms prometedor el uso de inhibidores de la ureasa
cico y con hipercalciuria idioptica. En stos probablemente la acidosis con cidos hidroxmicos.stos son molculas anlogas a la urea que forman
tubular no juegue un papel importante y responden a tiazidas. un complejo enzima-inhibidor irreversible. Se utilizan bsicamente dos sus
Otras circunstancias que favorecen la litiasis clcica: sarcoidosis, tancias de esta naturaleza: el cido propinico y el acetohidroxmico. Su em
sndrome de Cushing, diuresis escasa, dficit de inhibidores o anomalas pleo suele venir acompaado de cefaleas, temblores, trombosis venosas u
en el pH urinario (alcalosis). otros sntomas neurolgicos, por lo que tampoco son de gran aceptacin.
Litiasis clcica idioptica. Aproximadamente en el 80% de los pacien
tes con litiasis clcica no se demuestra ninguna anomala en el estudio Todo lo relativo al estudio de la nefrolitiasis expuesto anteriormente se pue
metablico. de repasar en la Tabla 6.

L i t i a s i s rica Tratamiento de la litiasis ya formada (Figura 7)


El cido rico no disociado es poco soluble en orina.Con un pH urinario de 5, Los clculos ya formados no expulsables (> 4-5 mm) precisan de tratamien
la solubilidad del cido rico es nicamente de 100 mg/1, mientras que con un to "agresivo es decir, necesitan ser extrados quirrgicamente o fragmenta
pH de 7 es de 1 .580 mg/1. Esto demuestra la gran importancia del pH urinario dos de forma que puedan ser expulsados espontneamente.
en la formacin de clculos de cido rico. Aparte de stos, tambin existe
una pequea proporcin de clculos de urato monosdico y urato amnico. A continuacin se analizan brevemente las diversas formas de tratamiento:
Ciruga abierta. Ha sido el tratamiento estndar hasta la aparicin de la
El objetivo del tratamiento es reducir el cido rico excretado y aumentar el litotricia extracorprea. An hoy, es preciso recurrir a la ciruga cuando
pH urinario (MIR 03-04, 80), ya que los clculos ms frecuentes en pacientes fracasan las ondas de choque o en determinados casos para reducir la
hiperuricmicos son los de cido rico. Por otra parte, este tipo de clculos masa litisica (clculos coraliformes).

0 3 U r o litiasis
Urologa I O3
Estruvita
Sales clcicas cido rico Cistina
(fosfato amnico magnsico)

Frecuencia Oxalato clcico: 55-60% 10-15% 5-10% 1-3%


Fosfato clcico: 1 0 -15%

Sexo Varn Mujer Varn Varn = Mujer

Etiologa Hipercalciuria idioptica Infeccin por grmenes ureasa (+) Gota (50%) Cistinuria
ldioptica ldioptica (< 50%)
Hiperuricosuria (20%) Hiperuricemias secundarias
pH Alcalino Alcalino cido cido
Radiologa Radioopacos Radioopacos Radiotransparentes Radiolcidos
Morfologa Formas prismticas polimrficas Aglomerados de cristales Cristales hexagonales en prismas
J/
')
{}o rffi
de los Cristales en atad" desorganizados. a veces o lminas
cristales formando masas continuas

Cristales de fostato Ca
g -$
$>
e:> fJJ
ql
IJJ
Wo
o
Cristales de oxea Cristales de estruvita Cristales de cido rico Cristales de cistina

Tratamiento Hipercalciuria idioptica: cido propinico y cido Alcalinizar la orina Forzar diuresis (ingesta hdrica)
tiazidas acetohidroxmico Alopurinol Alcalinizar orina
Hiperoxaluria 1 .1a: piridoxina Antibioterapia (si hay hiperuricemia) D-penicilamina
Hiperoxaluria 2.1a: En ocasiones ciruga Dieta de bajo contenido (si no hay respuesta)
colestiramina proteico
Tabla 6. Tabla-resumen de las nefrolitiasis

Manejo de la urolitiasis

j
Crisis
j
Estable
cuadro agudo cuadro crnico

Indican Edad?
Tipo de clculos?
o no LEOC
Periodicidad de la clnica?
o ciruga
No complicado Complicado Tipo de sntomas?
Viabilidad renal?

Tratamiento sintomtico: Obstruccin grave


Espasmolticos Infeccin, fiebre
y anti inflamatorios Dolor incoercible Tratar la condicin
Litotricia Ciruga

! ! !
Reposo e hidratacin Rin nico preexistente

t
Ecografa renal

t
Dilatacin
Clcica: acidificar (no til si oxalato),
citratos, tiazidas colestiramina
Extracorprea
(LEOC)
Ureterotoma
Pielolitotoma
y dieta baja en grasas y rica Percutnea Nefrectoma
Ingreso y tratamiento agresivo: en calcio, si hiperoxaluria ultrasonogrfica
Drenaje rica: alcalinizar, alopurinol con microlumbotoma
(catter o nefrostomfa) Estruvita: acetohidroxmico Endoscpica
Tratamiento parenteral: Cistina: D-penicilamina, vit. 86 va ureteral
- Antibiticos y alcalinizar
- Remontar hemodinmica Contraindicada en:
- Equilibrio electroltico Embarazo
- Narcticos Infeccin
Obstruccin distal
Vigilancia estrecha Aneurismas
Coagulopatas
Obesidad
Arritmia cardaca

Figura 7. Manejo de la litiasis renal ya formada (MIR 1 0- 1 1 , 99)


Manual CTO de Medicina y Ciruga, 9. edicin

Endourologa. La manipulacin endoscpica de la va urinaria es cada RECUERDA


da ms accesible gracias a las mejoras tcnicas. Puede realizarse extrac
cin directa del clculo mediante distintos tipos de pinzas o cestillas, o La endourologa y la litotricia extracorprea por ondas de choque
(LEOC) son los mtodos resolutivos para las litiasis no expulsables
bien fragmentar previamente el clculo mediante diversas fuentes de (> 4-5 mm); la ciruga abierta es hoy en dfa anecdtica para este fin.
energa, como la electrohidrulica, ultrasnica o lser. Se puede acceder
hasta el clculo mediante ureterorrenoscopia (URS), en los dos tercios
inferiores del urter; o nefrolitotoma percutnea (NLPC), en las litiasis Com pl icaciones
renales > 2 cm.
Litotricia extracorprea por ondas de choque (LEOC). Las ondas de La expulsin d e fragmentos litisicos puede ocasionar un clico renal y, con
choque se transmiten a travs de los tejidos corporales con la misma menor frecuencia, obstruccin ureteral (steinstrasse o "calle litisica").
impedancia acstica que el agua hasta alcanzar la litiasis, sobre la que
produce fenmenos de compresin y descompresin que conducirn Esta posibilidad es mayor ante litiasis de gran tamao, por lo que en algunos
a su fragmentacin. Prcticamente todos los clculos son susceptibles de estos casos se puede colocar un catter de derivacin urinaria (nefrosto
de tratamiento mediante LEOC. La nica limitacin seran aquellos cl ma o doble J) antes de la LEOC para disminuir este riesgo, generalmente en
culos no localizables por su pequeo tamao (< 2-5 mm). Cualquier litiasis superiores a 2 cm.
litiasis podra ser tratada con LEOC, aunque esto tendr que ser matiza
do en funcin de su tamao, composicin y dureza, localizacin, parti Derivadas del efecto directo de las ondas de choque, pueden aparecer con
cularidades anatmicas de la va excretora y paciente, funcin renal y tusiones renales manifestadas como hematuria, hematomas renales, equi
tipo de litotriptor disponible. La presencia de hipertensin arterial no mosis o eritema cutneo, y en grado mximo, rotura renal. La hematuria se
controlada facilita el riesgo de hemorragia durante la sesin de litotri considera la complicacin ms frecuente de la litotricia.
cia, luego deber ser estabilizada previamente a la misma y constituye,
en cierto modo, por ello, contraindicacin relativa de LEOC (Tabla 7) Ms controvertida es la terica relacin de la LEOC con la aparicin de hiper
(MIR 08-09, 93). tensin arterial, ya que no est demostrada en las ltimas revisiones publica
das, aunque s la relacin entre hematoma renal post-LEO( e hipertensin
arterial.
Relativas
Absolutas
(precisan de control previo al tratamiento)
Embarazo Alteraciones de la coagulacin ./ MIR 1 3-14, 21, 22
Obstruccin distal Aneurisma artico ./ MIR 10-1 1, 99, 1 1 6
Infeccin activa Alteraciones del ritmo cardaco, marca pasos ./ MIR 08-09, 93
o desfibriladores ./ MIR 06-07, 93, 106
Obesidad ./ MIR 05-06, 104
Hipertensin arterial descontrolada ./ MIR 04-05, 104
./ MIR 03-04, 44, 80
Tabla 7. Contraindicaciones de LEOC

" Precipitan en medio cido: cido rico y cistina. Precipitan en me


I d e a s c l a v e ,'S dio alcalino las que contienen fosfatos (fosfato amnico magnsico
o estruvita, y el fosfato clcico).
" Los clculos ms frecuentes son los de oxalato clcico.
" En el tratamiento de la litiasis por cido rico es beneficioso alcali
" Globalmente, la litiasis es ms comn en el varn, salvo las de estru nizar la orina.
vita, ms comunes en mujeres.
" Los clculos de oxalato NO se ven alterados por el pH (al Oxal, el pH
" La radiografa de abdomen no permite ver algunos clculos, como le da igual).
los de urato. Sin embargo, la ecografa puede verlos, independien
temente de su composicin. " Los clculos de estruvita se relacionan con microorganismos pro
ductores de ureasa, como Proteus.
" Litiasis radiotransparentes: Sulfamidas, lndinavir, Urato, Xantinas
(SIUX). Las de cistina son radiolcidas; y el resto, radioopacas. " Las contraindicaciones absolutas para LEOC son: embarazo, infec
cin activa y obstruccin de las vas urinarias distal al clculo.
" Las tiazidas son tiles para la hipercalciuria idioptica.

" Los clculos asociados a las resecciones ileales o a la enfermedad


inflamatoria intestinal son de oxalato clcico.

03 U r o l i t i a s i s
-11
F
Urologa I O3
dolor en fosa renal izquierda de 5 das de evolucin, asociado en
Casos clnicos ., / las ltimas 24 horas a fiebre, escalofros y malestar general. Anal
tica de sangre: plaquetopenia, leucocitosis y disminucin de la ac
A un hombre de 29 aos, con antecedentes de dolor tipo clico tividad de la protrombina. Analtica de orina normal. Radiografa
en fosa renal izquierda que cedi con tratamiento analgsico, se de abdomen con claras imgenes de litiasis. Eco renal: dilatacin
le practica una urografa intravenosa, aprecindose defecto de re moderada de sistema excretor izquierdo. Cul es la conducta ms
plecin radiotransparente de 6 x 7 mm en tercio distal de urter iz adecuada?
quierdo. El pH de la orina fue de 5,5; asimismo, se observan cristales
de urato, 9-12 hemates por campo y escasa leucocituria. Cul sera 1) Solicitar hemocultivos y urocultivo para establecer la necesidad de
el tratamiento ms apropiado? antibioterapia.
2) Realizar urografa intravenosa para intentar filiar la causa.
1) Alopurinol va oral. 3) Hidratar a la paciente bajo observacin rigurosa, y repetir ecografa
2) Ureteroscopia con extraccin del clculo. a las 48 horas.
3) Nefrolitotoma endoscpica percutnea. 4) Colocar catter doble J o practicar nefrostoma percutnea de for
4) Alcalinizacin de la orina por va oral. ma inmediata con cobertura antibitica.
S) Administracin de D-penicilamina. 5) Laparotoma exploradora para objetivar causa, y realizar tratamien
to antes de que el cuadro est muy evolucionado.
RC: 4; MIR 03-04, 80
RC: 4
Mujer de 50 aos, diabtica insulinodependiente, con infecciones
urinarias y clicos nefrticos de repeticin. Acude a Urgencias por
Urologa

TUMORES RENALES

El adenocarcinoma renal es un tema Entre los factores de riesgo que se han implicado se encuentran el humo
"de moda" en el examen MIR. Cualquier del tabaco, el cadmio y la obesidad. Existen formas familiares que suelen ser
aspecto de este tema puede aparecer, mltiples y bilaterales, como en la enfermedad de Von Hippel-Lindau y, en
pero es fundamental reconocer menor medida, la esclerosis tuberosa. Se han identificado alteraciones cro
el sndrome de Stauffer, que ha sido mosmicas que implican al cromosoma 3.
recientemente introducido y preguntado
varias veces consecutivas.
Asimismo, existe una incidencia aumentada en el rin poliqustico, en la en
fermedad qustica adquirida de la insuficiencia renal crnica (MIR 07-08, 1 03)
y en los riones malformados, como el rin "en herradura''. Procede de las
4.1 . Carcinoma de clulas renales clulas del tbulo contorneado proximal, y microscpicamente predomi
nan las clulas claras sobre las granulares y fusiformes.
(adenocarcinoma renal, hipernefroma)
Es el tumor slido renal ms frecuente (90%) (Figura 8) (MIR 1 0-1 1 , 24). Se RECUERDA
trata de un tumor fundamentalmente de la edad adulta, con mayor inciden
La esclerosis tuberosa y la enfermedad de Von Hippel-Lindau se aso
cia entre los 40-60 aos, con predominio en el varn (2:1) a excepcin de la cian tambin a otro tumor renal, el angiomiolipoma.
variedad cromfoba, tpica de las mujeres. El ms frecuente es el tumor de
clulas claras, aunque hay otras muchas variedades (papilar, cromfobo . . . )
(MIR 1 2-1 3, 1 27). Presentacin
La trada clsica (hematuria, dolor y masa en el lanco) ocurre nicamente en
el 10% de los casos y, cuando se presenta as, generalmente se trata de una
enfermedad avanzada (MIR 08-09, 95).

El 30% presenta metstasis a distancia en el momento del diagnstico aun


que, contrariamente, cada vez son ms los hallazgos incidentales al realizar
ecografas abdominales de rutina por otra causa, alcanzando en algunos es
tudios ms de la mitad de los casos diagnosticados.

La anomala ms frecuente es la hematuria macroscpica o microscpica


(60%). Otros hallazgos frecuentes son dolor (40%), prdida de peso (30%),
anemia (40%), masa en lanco (24%), hipertensin arterial (20%), hipercalce
mia (6%), eritrocitosis (3%). El 20% de los pacientes presentan como cuadro
paraneoplsico alteracin de las enzimas hepticas sin evidencia de metsta
sis (sndrome de Stauffer) (MIR 1 3-14, 1 26; MIR 09-1 O, 99; MIR 06-07, 1 02;
MIR 05-06, 1 05).

Ocasionalmente, el adenocarcinoma renal puede producir hormonas pro


ductoras de sndromes clnicos segn la sustancia secretada. Entre stas se
encuentran pptidos PTH-/ike, prostaglandinas, prolactina, renina, gonadotro
pinas o corticoides. La invasin de la vena renal principal puede ocasionar la
Figura 8. Carcinoma de clulas renales aparicin de un varicocele de forma repentina, que no disminuye en decbito.

fl
Urologa I O4
RE C U ERDA La resonancia magntica (RM), aunque no se emplea de manera rutinaria
en este tipo de pacientes, s se utiliza como mtodo de diagnstico bsico
La produccin de pptidos puede aparecer en el hipernefroma, pero
es ms tpica de carcinomas epidermoides (pulmn, esfago. . . ). en sujetos en los que se sospecha afectacin trombtica tumoral de la vena
renal o cava (Figura 1 1 ) (MIR 04-05, 1 05).

Diagnstico
La ecografa es la primera prueba complementaria que debe realizarse, de
forma que la identificacin ecogrfica de una lesin que cumple criterios
de quiste simple (contorno liso, contenido transnico y refuerzo posterior)
hace innecesarios mayores esfuerzos diagnsticos, pudiendo efectuarse un
seguimiento ecogrfico anual. De esta forma se diagnostican la mayora de
las masas renales en la actualidad (Figura 9).

Figura 1 1 . RM de tumor renal con trombo en venas renal y cava

Como en la prctica totalidad de la patologa urolgica, la UIV est siendo


cada vez ms desplazada por las distintas modalidades de estudio con TC,
pero adems en el adenocarcinoma renal proporciona pocos datos y de for
ma indirecta, como puede ser la distorsin del sistema colector, su ocupa
cin o la anulacin funcional del rin. En las placas tomogrficas de la UIV
puede observarse la presencia de una masa o una alteracin del contorno
Figura 9. Ecografa de quistes renales simples renal.

La realizacin de puncin-aspiracin con aguja fina (PAAF) de una masa re La arteriografa renal, exploracin obligada hace aos, ha quedado relegada
nal para su filiacin es una exploracin agresiva que, debido a su baja sensi a los casos dudosos, riones nicos y otras situaciones en las que se plantea
bilidad, no se justifica actualmente, excepto en casos excepcionales. tratamiento quirrgico conservador. El patrn arteriogrfico caracterstico
incluye neovascularizacin tumoral, lagos venosos, fstulas arteriovenosas y
La TC es el mejor mtodo aislado para evaluar una masa renal; proporciona vasos capsulares.
informacin precisa sobre metstasis ganglionares (80%) y afectacin de r
ganos adyacentes (Figura 1 O). El estudio de extensin, si se sospechan metstasis, se completar realizan
do radiografa de trax, analtica heptica completa y, en algunos casos du
dosos, gammagrafa sea.

El procedimiento diagnstico ante la presencia de masas renales se puede


observar en la Figura 1 2.

Tratamiento
Una vez estudiado el tumor y descartada la presencia de metstasis, tan
to viscerales como ganglionares, el tratamiento de eleccin es la nefrec
toma radical, incluyendo la fascia de Gerota y la glndula suprarrenal
(MIR 10-1 1 , 23).

En ocasiones especiales se puede plantear la ciruga conservadora o parcial,


como en los tumores bilaterales, en aqullos que aparecen sobre rin ni
co, o sujetos con nefropatas mdicas, en los que la prdida de masa nefro
Figura 1 o. TC de masa renal izquierda nal obligara a dilisis.
Manual CTO de Medicina y Ciruga, 9. edicin

Examen fsico
. Anlisis de orina
l
Masa renal descubierta accidentalmente titirosincinasas (sunitinib, bevacizumab) en primera lnea, antiangiognicos
en segunda lnea. Las situaciones que favorecen la respuesta al tratamiento
inmunoteraputico son presencia de metstasis pulmonares exclusivamen
te, buen estado general, y que se haya realizado la nefrectoma antes del

MihifiM
descubrimiento de las masas pulmonares.

/ En la actualidad se investiga sobre autovacunas elaboradas con linfocitos


peritumorales que parecen ofrecer resultados alentadores.
Quiste complejo
Quiste simple
o masa slida

4.2. Otros tu mores


Observar

Tumor de Wilms (vase Seccin de Pediatra).


Tumores renales metastsicos. Pueden encontrarse metstasis en el
Masa slida
o quiste complicado
Angiomiolipoma rin de tumores de pulmn (lo ms frecuente), mama, melanomas e
infiltracin por linfoma (MIR 1 1 -12, 1 09).
Tumores benignos:
No complicado: Complicado:
Adenomas corticales. Son los tumores ms frecuentes del adulto,
Nefrectoma
radical observacin Nefrectoma simple aunque indistinguibles clnicamente del adenocarcinoma, por lo
o parcial Nefrectoma parcial que se tratan como tales. El criterio clsico de tamao (3 cm) para
Embolizacin su diagnstico diferencial no es vlido en la actualidad.
--- - - --
Figura 1 2. Algoritmo diagnstico de las masas renales Angiomiolipomas. Se asocian a la esclerosis tuberosa en un 50%.
Compuestos de una proporcin variable de grasa, vasos y fibras
Adems de en estas indicaciones imperativas, actualmente es el nuevo patrn musculares. Cuando son grandes (mayores de 4 cm), pueden oca
de referencia en el tratamiento de tumores pequeos (menos de 4 cm), bien sionar un sndrome de Wnderlich por sangrado retroperitoneal.
delimitados y sin afectacin de la grasa perirrenal. En estos pacientes seleccio Cuando se asocian a esclerosis tuberosa, suelen ser mltiples y bila
nados parece que la supervivencia y la tasa de recidivas locales son semejan terales, por lo que deben tratarse de forma conservadora.
tes a las que se presentan en casos similares tratados con nefrectoma radical. Oncocitoma. Considerado benigno, aunque en algunos se han
detectado metstasis. Hay criterios radiolgicos para distinguirlo
La linfadenectoma regional no mejora la supervivencia y nicamente tiene del adenocarcinoma, pero en la mayora de los casos, ni stos ni la
validez, por tanto, en la estadificacin, por lo que no se realiza sistemtica citologa o la biopsia ofrecen garantas suficientes de su benigni
mente. Se establece sobre la base de los hallazgos quirrgicos y anatomo dad, por lo que tienden a ser tratados mediante nefrectoma.
patolgicos. Nefroma mesoblstico (hamartoma fetal). Es el tumor benigno
ms frecuente en recin nacidos y lactantes.
Otras formas de tratamiento carecen de eficacia. Tanto la quimioterapia
como la radioterapia ofrecen resultados pobres. En el caso de enfermedad
metastsica, las opciones son mltiples, pero ninguna satisfactoria. Aunque ./ MIR 1 3-14, 126
se ha descrito la regresin de las lesiones metastsicas tras la nefrectoma, ./ MIR 1 2-1 3, 1 27
esto ocurre nicamente en un 1 %, y generalmente de forma transitoria, por ./ MJR l l -1 2, 1 09
lo que no se justifica, salvo de forma paliativa por otros motivos. ./ MIR 10-1 1 , 23, 24
./ MIR 09-1 O, 99
./ MIR 08-09, 95
La inmunoterapia con interferones, interleucina, linfocitos killer activados y ./ MIR 07-08, 103
ciertas combinaciones de quimioterapia con inmunoterapia son alternativas ./ MIR 06-07, 102
para la enfermedad metastsica, pero en ninguna de ellas se obtienen tasas ./ MIR 05-06, 105
de respuesta superiores al 1 5%. Actualmente se utilizan de preferencia an- ./ MIR 04-05, 105

'9 Hay que sospechar tumor renal ante un varicocele izquierdo, de


I de a s clave fa::{ aparicin sbita y que no cede con el decbito.

'9 El ms frecuente de los tumores slidos renales es el hipernefro '9 El hipernefroma puede producir multitud de sndromes paraneo
ma. plsicos. Esto puede complicar bastante el diagnstico; de ah el
sobrenombre de "tumor del internista''.
'9 El paciente caracterstico es un varn de mediana edad, obeso y fu
mador. '9 La elevacin de las transaminasas sin afectacin heptica es tpica
del hipernefroma (sndrome de Stauffer).
'9 La trada clsica consiste en hematuria, dolor y masa en flanco.
Actualmente, lo ms habitual es que sea incidentaloma (asin '9 No confundir un quiste simple con un hipernefroma. Los criterios
tomtico). Si produce sntomas, el ms frecuente es la hema de quiste simple son contorno liso, contenido transnico y refuer
turia. zo posterior.

04 Tu mores ren ales


Urologa I O4
" La primera prueba de imagen, ante la sospecha de hipernefroma, '9 El tratamiento fundamental del h ipernefroma es la extirpacin qui
sera la ecografa. rrgica. La quimioterapia y radioterapia tienen un papel muy se
cundario.

Un paciente de 62 aos, con alteracin de la funcin renal y crisis


Casos clnicos .. de hematuria, presenta una masa abdominal palpable en flanco
derecho. Se le realiza una TC, detectndose una masa de carc
Un hombre de 45 aos tiene un carcinoma de clulas renales extendido. ter slido de 8 cm de dimetro en rin derecho. En la anamne
Los niveles de GOT, fosfatasa alcalina, LDH y a-2 globulina son elevados sis destaca que el paciente es fumador de 35 cigarrillos al da.
y el tiempo de protrombina alargado. El hgado aparece difusamente Cul es, entre los siguientes, el diagnstico de presuncin ms
agrandado, pero no existen defectos focales de infiltracin intrahep probable?
tica. La explicacin etiolgica ms probable para estos hallazgos ser:
1) Nefroblastoma.
1) Efectos hepatotxicos de tumor. 2) Liposarcoma.
2) Metstasis heptica. 3) Angiomiolipoma.
3) Amiloidosis. 4) Adenocarcinoma.
4) Trombosis tumorales que obstruyen la vena heptica. 5) Carcinoma epidermoide.
S) Hepatitis vrica aguda.
RC: 4
RC: 1; MIR 05-06, 105
Urologa

H IPERPLASIA BENIGNA DE PRSTATA


Y CARCINOMA PROSTATICO

O R I E N TA C I N La hiperplasia prosttica benigna y el cncer de prstata son dos temas fundamentales. Probablemente

M I R el cncer sea ms importante, sobre todo en lo referente al tratamiento. Hay que aprenderse muy bien
el resumen de la Tabla 9; aporta muchas preguntas acertadas y requiere poco esfuerzo.

5.1 . H iperplasia prosttica benigna

La hiperplasia prosttica benigna (HPB) afecta en mayor o menor grado a la


gran mayora de los varones a partir de la quinta dcada de la vida, alcanzan Zona de transicin
do el 80-95% de la poblacin masculina de 80 aos.

La prstata se divide clsicamente en cinco lbulos (anterior, medio, poste


rior y dos laterales), aunque stos nicamente se encuentran como tales en la
edad fetal.En el adulto se puede interpretar la anatoma de la prstata dividida
en dos partes: una zona perifrica, donde se origina principalmente el carcino
ma, y una zona periuretral o transicional, de la que procede la HPB (Figura 1 3).

La HPB est compuesta de una proliferacin variable de elementos glandu


lares, musculares y del estroma, que en su crecimiento comprimen la prs Lbulo anterior
tata perifrica, formando la llamada cpsula quirrgica. Su etiopatogenia no
est clara; aunque el estmulo andrognico a travs de su forma activa, la di
hidrotestosterona, es fundamental, su papel exacto no ha sido determinado.
Las teoras ms recientes abogan por un desequilibrio hormonal de estrge
nos/andrgenos, o por la existencia de factores de crecimiento prostticos
con un papel permisivo del ambiente hormonal.

No existe evidencia de asociacin entre HPB y carcinoma prosttico (MIR 13-14,


127).

Diagnstico
Lbulo Lbulo posterior
El crecimiento prosttico se produce generalmente hacia la uretra, lateral
ocasionando obstruccin de sta y dificultando el vaciamiento vesical
Glndulas
(Figura 1 4) . suburetrales

Esto no se manifiesta inmediatamente, sino que, habitualmente, el proceso


pasa por una serie de etapas que incluyen una fase de compensacin, una
clnica y una de descompensacin: Figura 1 3. Anatoma de la prstata

-11-
p
Urologa I O5

Figura 14. Ecografa de hiperplasia prosttica con crecimiento


del lbulo medio intravesical

1. Fase d e compensacin. E l crecimiento prosttico ocasiona u n aumen


to de la presin uretral durante el vaciado que es compensado por una
Figura 16. Hiperplasia prosttica en fase de descompensacin. Situacin
mayor actividad contrctil del detrusor que se hipertrofia, encontrando
del mismo paciente un ao despus de la anterior: hidronefrosis grave.
presiones vesicales ms elevadas. En esta fase, la clnica puede ser mni De la vejiga (no se observa) se evacuaron 3.500 mi de orina
ma o inexistente.
2. Fase clnica (Figura 15). La elongacin de las fibras musculares por en Pueden producirse tambin otro tipo de sntomas denominados "irritativos
cima de un lmite condiciona prdida de capacidad contrctil. En este que son debidos a la alteracin funcional vesical y cuya resolucin es ms
momento aparece retraso del inicio de la miccin, disminucin del ca difcil tras la desaparicin de la obstruccin. Entre estos sntomas se inclu
libre y de la fuerza del chorro miccional y alargamiento del vaciado (lo yen polaquiuria, tenesmo, nicturia y urgencia miccional (MIR 10-1 1 , 100).
que en conjunto se denomina sndrome prosttico). El vaciado suele ser La HPB es la causa ms frecuente de obstruccin del tracto urinario inferior
incompleto, dando lugar a un residuo posmiccional. en el varn.

En la evaluacin del sndrome prosttico, el tacto rectal contina siendo


la exploracin fundamental, sobre todo para diferenciarlo del carcinoma,
ya que no es infrecuente que ambas entidades coexistan. La clnica es lo
ms importante para valorar la indicacin de tratamiento de la HPB, ya
que no existe correlacin entre el tamao prosttico y el grado d e obs
truccin (MIR 08-09, 1 06). Cualquier zona sospechosa al tacto debe ser
biopsiada.

La medicin del flujo mximo miccional es tambin importante, conside


rndose normal cuando es mayor de 1 5 ml/s y claramente patolgico si
es menor de 1 0 ml/s. El estudio puede completarse con una ecografa que
permita evaluar si existe afectacin del tracto urinario superior, residuo pos
miccional, litiasis vesical u otra patologa asociada. El uso del PSA en la HPB
nicamente est indicado para descartar la presencia de carcinoma en la
prstata, ya que no sirve para diagnosticar HPB, aunque recientemente ha
demostrado ser el mejor predictor de la historia natural de la enfermedad. Es
decir, que mayores niveles de antgeno prosttico especifco (prostate-spe
cific antigen, PSA) en HPB diagnosticada probablemente se correlacionarn
con mayores volmenes prostticos y con ms posibilidades de complica
Figura 1 S. Hiperplasia prosttica en fase clnica. UIV mostrando impronta cin derivadas de la HPB.
prosttica en vejiga (vejiga "en montera") con urteres en anzuelo
Tratamiento
3. Fase de descompensacin (Figura 16). Se produce un vencimiento
del detrusor vesical, que es incapaz de vencer la presin uretral, aumen Dentro de las posibilidades teraputicas, la ciruga contina siendo el ni
tando la sintomatologa anterior y pudiendo presentarse retencin uri co tratamiento definitivo para la HPB. sta puede ser endoscpica (RTUP:
naria. Ocasionalmente puede aparecer dilatacin ureteral bilateral con reseccin transuretral prosttica) o abierta (adenomectoma prosttica)

---
deterioro de la funcin renal. Esto se debe a uropata obstructiva infra (Figura 1 7), dependiendo del tamao del adenoma. En el 10% de las piezas
vesical con prdida del mecanismo antirreflujo. obtenidas se encontrarn focos de adenocarcinoma incidental.
Manual CTO de Medicina y Ciruga, 9. edicin

5.2. Carcinoma prosttico

El adenocarcinoma prosttico es el tumor maligno ms frecuente del apara


to genitourinario masculino y el segundo en frecuencia general, despus del
pulmonar.Sin embargo, si se incluyesen los carcinomas incidentales y los en
contrados en autopsia, supera al pulmonar en prevalencia (MIR 06-07, 1 03).

La hormonodependencia del cncer prosttico parece indicar el papel de


los andrgenos en su etiologa o patogenia. La relacin de factores genti
cos, ambientales o infecciosos no ha quedado suficientemente establecida.

El 95% de los carcinomas prostticos son adenocarcinomas originados en


la zona perifrica de la prstata. Los carcinomas ductales se originan en los
conductos prostticos en lugar de los acinos, e histolgicamente pueden
corresponder a carcinomas transicionales, escamosos, endometrioides o
Figura 17. Adenomectoma prosttica mixtos. Ms raros son los carcinosarcomas (menos del 1 %).

Se debe tener en cuenta que en la ciruga de la HPB no se extirpa la cpsula El adenocarcinoma prosttico, con frecuencia, es multifocal y presenta po
quirrgica, que est constituida por las glndulas prostticas perifricas com blaciones en distinto grado de diferenciacin. En esta heterogeneidad se
primidas por el adenoma, y es el principal origen del carcinoma prosttico, por basa la clasificacin de Gleason, que asigna una puntuacin de 1 a 5, segn
lo que la intervencin quirrgica no protege del desarrollo de este proceso. el patrn histolgico de cada una de las dos poblaciones ms representa
tivas de la masa, sumando ambas puntuaciones para obtener un resultado
Los tratamientos no quirrgicos incluyen una variedad de fitoterapias, final de 2 a 1 O. La escala de Gleason se corresponde con el pronstico de
poco efectivas si se valoran con parmetros objetivos, inhibidores de la la enfermedad, independientemente del estadio (MIR 06-07, 233). Para
5a-reductasa (finasterida, dutasterida) (MIR 04-05, 225) que reducen el ta la estadificacin se emplea principalmente la clasificacin TNM (Tabla 8
mao prosttico, antagonistas a-adrenrgicos (alfuzosina, prazosina, doxa y Figura 18).
zosina, terazosina, tamsulosina, etc.) que relajan la musculatura del cuello
vesical y uretra (MIR 1 2-1 3, 1 24). En principio, estos frmacos deben utilizar Clnica
se en escalada teraputica (es decir, iniciar monoterapia con a-bloqueante,
independientemente de la sintomatologa y del tamao prosttico, e ir su El carcinoma prosttico es una enfermedad ms frecuente en ancianos, y
biendo segn lo necesario) a pesar de la aceptacin de estudios como el la mayora de ellos se diagnostica por encima de los 60 aos. Clnicamente
COMBAT, que parece indicar que en pacientes con sintomatologa a partir puede producir sntomas obstructivos del tracto urinario inferior superpo
de moderada, y con volmenes prostticos por encima de 30-40 cm3, se nibles a los de la HPB. A stos puede aadirse la hematuria. El 25% de los
debe realizar de inicio terapia combinada. pacientes que refieren retencin urinaria aguda presentan un carcinoma
prosttico. Aproximadamente un 25% de los pacientes presentan metsta
sis en el momento del diagnstico; stas pueden producir manifestaciones
RECUER D A como dolor seo, compresin medular, mieloptisis o coagulopata. Por for
tuna, estos casos se encuentran en claro descenso gracias a la incorporacin
La finasterida tambin es til para la alopecia andrognica, donde
se emplea en dosis mucho menores. del PSA, que facilita el diagnstico de la enfermedad en estadios tempranos
y comnmente asintomticos.

Como inconvenientes principales de los inhibidores de la 5a-reductasa se Diagnstico


encuentran impotencia, reduccin del PSA en torno al 50% (dificultando el
diagnstico del carcinoma, si lo hubiese) y que tardan una media de 4 meses Ta cto rectal
en hacer efecto.
Contina siendo el mtodo fundamental de cribado.Son accesibles al tacto
De los a-bloqueantes, el principal inconveniente es la hipotensin. rectal todos los estadios excepto el Tl , que por definicin es un hallazgo.
Caractersticamente, el carcinoma es duro, nodular e irregular. En general, se
En cuanto a las indicaciones de tratamiento quirrgico, globalmente, slo un puede realizar un tacto rectal y un PSA anual a todos los varones por enci
10% de los pacientes prostticos precisar ciruga. La intensidad de las mani ma de 50 aos aunque, de momento, la Organizacin Mundial de la Salud
festaciones clnicas subjetivas y la mala respuesta al tratamiento mdico pue (OMS) no aconseja la realizacin de cribado poblacional sistemtico.
den constituir la indicacin para la intervencin. Entre las causas "objetivas"
que suponen indicacin absoluta de tratamiento quirrgico se encuentran: M a rcad ores t u m o r a l e s
Retencin urinaria reiterada.
Hidronefrosis retrgrada (lesin del parnquima renal por obstruccin S e dispone fundamentalmente de dos marcadores tumorales. La fosfatasa
infravesical). cida prosttica (FAP) se emplea en clnica desde hace dcadas; es un mar
Infeccin urinaria de repeticin. cador especfico, pero su elevacin suele indicar extensin extraprosttica,
Litiasis vesical. por lo que no resulta til en el diagnstico precoz. El PSA es realmente un
Hematuria de repeticin. marcador de tejido prosttico cuyos niveles suelen encontrarse ms ele-

0 5 H i per p lasia benigna de prs t a t a


y carcinoma pros t t ico
r
Urologa I O5
T: define el tumor
T1: tumor inaparente clnicamente (no palpable ni visible por tcnicas T4: tumor fijo o invade rganos adyacentes distintos a las vesculas
de imagen): seminales (cuello vesical, esfnter externo, recto, msculo elevador
- Tl a: hallado incidentalmente. Afectacin menor del 5% del tejido resecado o pared pelviana)
- Tl b: hallado incidentalmente. Afectacin mayor del 5% del tejido resecado N: define la afectacin ganglionar
- Tl e: tumor identificado por puncin-biopsia por aumento del PSA
NX: no se pueden estudiar los ganglios regionales
T2: tumor confinado a la prstata (incluye la invasin de la cpsula NO: no metstasis ganglionares
prosttica sin exteriorizacin del tumor hacia el tejido adiposo periprosttico): N 1: metstasis a ganglios regionales
- T2a: menos del 50% de un lbulo M: define las metstasis
- T2b: ms del 50% de un lbulo
MO: no metstasis
- T2c: dos lbulos
M1: metstasis a distancia:
T3: extensin del tumor por fuera de la cpsula: - M 1 a: ganglios linfticos no regionales
- T3a: extensin transcapsular (sea unilateral o bilateral) - Ml b: hueso
- M 1 e: otras localizaciones
- T3b: invasin de la(s) vescula(s) seminal(es)

Tabla 8. Estadificacin del carcinoma de prstata

T: tumor primario clnica TN T3a

T2a T2b T3
TX No puede evaluar el tumor
TO No existen signos de tumor primario

Tla Tlb
T2c

< 5% > 5%

T1 Tumor no evidente clnicamente, no palpable T2 Tumor limitado a la prstata T3 Tumor que se extiende a travs
ni visible mediante tcnicas de imagen: o a la cpsula, sin sobrepasarla: de la cpsula prosttica:
Tl a Extensin menor o igual al So/o del tejido T2a Menos del 50% de un lbulo T3a Extensin extracapsular (unilateral
resecado T2b Ms del 50% de un lbulo o bilateral)
T1 b Extensin mayor del So/o del tejido T2c Dos lbulos T3b Tumor invade la vescula seminal
resecado
T1 c Tumor identificado mediante puncin
bipsica (consecuencia de un PSA elevado) N: ganglios linfticos regionales

T4 Tumor fijo o que invade estructuras adyacentes diferentes a las vesculas seminales NX No se pueden evaluar los ganglios linfticos regionales
NO No hay metstasis ganglionares regionales
Nl Metstasis en ganglios linfticos regionales

Figura 18. Estadificacin del adenocarcinoma de prstata


Manual CTO de Medicina y Ciruga, 9. edicin

vados en el cncer, pero es inespecfico y tambin estn elevados a con La ecografa abdominal no tiene valor en la deteccin del carcinoma pros
secuencia de patologa benigna (infecciones, sondajes, HPB, etc.). Por este ttico. La TC y la RM tienen su principal papel en la estadificacin ganglio
motivo, se ha intentado aumentar su especificidad para cncer con otros nar y la valoracin de metstasis a distancia. Las primeras metstasis de
parmetros (densidad de PSA, ndice PSNedad, velocidad de cambio del ben buscarse a nivel de los ganglios linfticos de las cadenas obturatrices
PSA, PSA libre), aunque an no ha quedado establecida la ventaja de stos e ilacas.
sobre el PSA aislado.
G a m m agrafa sea
Si el PSA es menor de 4 ng/ml, es poco probable que se encuentre un cncer
de prstata.Si es mayor de 1 O, las probabilidades aumentan, lo que aconse Se utiliza para la deteccin de metstasis seas; tiene mayor sensibilidad que
jara una biopsia de prstata ecodirigida.Si est entre 4-1 O, se pueden utili la radiologa convencional (Figura 20), y debe realizarse en todo paciente
zar los parmetros antes mencionados para valorar la necesidad de biopsia. en el que se sospechen metstasis (Gleason 8 y/o PSA 20). Antes de
plantearse el tratamiento curativo, en ciertos pacientes con altas probabili
Pruebas d e image n dades de encontrarse el cncer extendido, se debe efectuar una gammagra
fa previa para confirmar la no existencia de metstasis seas y una TC para
La ecografa transrectal (ETR) (Figura 1 9 ) e s el mtodo de imagen ms til descartar metstasis ganglionares (estudio de extensin) (MIR 09-1 O, 98).
para la estadificacin local, pudiendo ofrecer informacin importante sobre
la afectacin capsular, de vesculas seminales, cuello vesical o recto.Aunque
no existe un patrn caracterstico, suele aparecer como ndulos hipoeco
gnicos. La ETR ofrece, adems, la posibilidad de dirigir la biopsia hacia las
zonas sospechosas.

Figura 20. Radiografa de columna. Metstasis osteoblsticas

RECUER D A
las metstasis del cncer de prstata son osteoblsticas, es decir,
forman hueso (se ven mejor en la radiografa simple, no en la gam
magrafa).

Biopsia prosttica

Debe realizarse para la confirmacin del diagnstico (MIR 1 3-14, 1 24). Pue
de efectuarse va transrectal o transperineal, guiada por el tacto rectal o por
la ETR, lo que aade efectividad a la prueba. La realizacin de la biopsia est
indicada siempre que exista una anomala del tacto rectal, elevacin de los
marcadores tumorales o alteracin en las pruebas de imagen. La PAAF es
una alternativa con menores complicaciones, pero con el inconveniente de
que no puede evaluar el grado histolgico (Gleason).

RECUER D A
Son indicaciones de biopsia prosttica el tacto rectal sospechoso, la
Figura 19. ETR de adenocarcinoma prosttico. (A) ETR corte transversal; presencia de un ndulo ecogrfico y un PSA > 4 (variable la cifra se
(B) ETR corte longitudinal; (C) adenocarcinoma, ndulo hipoecoico gn criterios).
en lbulo derecho

0 5 H i per plasia benigna de p r s t a t a


y c a r c inoma p r o s t t ico
p
Urologa I O5
Tratamiento Trata m i e n t o p o r estadios (MIR 04-05, 1 06)

O p c i o n e s tera puticas Estadio Tl a. Tiene una mortalidad por la enfermedad del 2% a los 1 O
aos, por lo que no precisa tratamiento, salvo quiz los pacientes jve
Prostatectoma radical. Los pacientes candidatos deben ser indivi nes (menores de 60 aos) con una elevada esperanza de vida.
duos con una esperanza de vida superior a 1 O aos.Como complica Estadio Tl b-T1 c. Alcanzan una mortalidad del 80% dejados a su evolucin
ciones, se puede encontrar incontinencia (2-57%) (MIR 1 1 - 1 2, 1 1 O), natural. Por ello est indicada la prostatectoma radical, la radioterapia ex
estenosis anastomtica (1 0%), impotencia (50%) o incluso la muerte terna o braquiterapia, en sujetos con esperanza de vida superior a 1 O aos.
(< 5%). En lneas generales, suele ir acompaada de linfadenectoma Estadio T2a. Es la indicacin ms clara de prostatectoma radical. La ra
leo-obturatriz (salvo en los pacientes con tumores considerados de dioterapia o braquiterapia se reservara para pacientes de riesgo quirrgi
bajo riesgo). co elevado o que no aceptan efectos secundarios atribuibles a la ciruga.
Radioterapia. Como tratamiento curativo, los resultados en estadios Estadio T2b y T2c. Un 40% demuestra ser en realidad estadio 3, tras el
localizados se acercan a los de la ciruga. La diarrea crnica, la procti anlisis de la pieza quirrgica de prostatectoma radical (infraestadifica
tis, la cistitis rdica y las fstulas urinarias son complicaciones del tra cin). La radioterapia externa o braquiterapia tambin puede ser til en
tamiento, as como la incontinencia y la impotencia a partir de los pacientes de alto riesgo quirrgico.
2 aos de tratamiento. Se ha empleado tambin radioterapia inters Estadio T3a. La indicacin quirrgica es dudosa, as como la radiotera
ticial (braquiterapia) con implantacin de yodo- 1 23 (1-1 23), oro-1 98 pia local, por lo que solamente se propondra a sujetos jvenes, aun a
(Au- 1 98), paladio e iridio. Su indicacin queda limitada a tumores pe costa de obtener malos resultados. Generalmente son tratados como el
queos de estadio Tl o T2, y sus resultados son similares a los de la grupo siguiente.
ciruga. En caso de compresin medular o dolor por metstasis seas, Estadio T3b, T4, N+, M+. Varn aoso con mal estado general. El
la radioterapia sobre la metstasis puede conseguir el control local de tratamiento hormonal es la opcin indicada. Puede ser preciso el
la enfermedad. uso de radioterapia paliativa sobre la metstasis en caso de dolor
Hormonoterapia. El adenocarcinoma prosttico est compuesto por (MIR 03-04, 9 1 ).
una poblacin heterognea de clulas androgenodependientes y an
drogenoindependientes. La supresin hormonal frena el crecimiento Recidiva t u m o r a l posterior
de las primeras, pero no afecta a las androgenoindependientes. Se pue a trata m i e n t o c o n intencin c u rativa
de conseguir disminuir los niveles de andrgenos circulares por distin
tos mtodos:
Castracin quirrgica. Es el mtodo aislado ms eficiente, con la Despus de la realizacin de prostatectoma radical, los pacientes son mo
ventaja de que elimina la necesidad de medicacin permanente. nitorizados generalmente con peticiones de PSA. Cuando las cifras de PSA
Por su rapidez en el efecto supresor hormonal, tambin est indica tras prostatectoma radical son superiores a 0,2 ng/ml, se considera recidiva
da en las compresiones medulares por metstasis. bioqumica y debe hacer sospechar la existencia de metstasis a distancia, o
Estrgenos (dietilestilbestrol). Inhiben la secrecin de LH. Ac bien la existencia de recidiva a nivel local (MIR 05-06, 106).
tualmente este mtodo se ha abandonado debido al alto riesgo
cardiovascular que conlleva.
Progestgenos. Inhiben la secrecin de LH y actan como anti Adenocarcinoma
andrgenos, al unirse a los receptores de la dihidrotestosterona. Es prosttico
preciso aadir estrgenos para evitar el fenmeno de escape, que Localizacin Zona transicional Perifrica
se produce tras varios meses de tratamiento. No son de uso habi
Clnica Fases: Mayora asintomticos:
tual.
Compensacin Hasta 25%. Sndrome
Agonistas LHRH. Aunque inicialmente ocasionan un aumento
Clnica prosttico
de los niveles de testosterona, posteriormente suprimen la se
Descompensacin Hasta 25%. Retencin aguda
crecin de LH y de andrgenos. La elevacin transitoria de los
Hasta 25%.Metstasis
andrgenos puede empeorar el cuadro clnico, principalmente
si existe compromiso medular por metstasis seas. Esta eleva Diagnstico Tacto rectal
cin (flare-up) se debe suprimir mediante la administracin de diferencial ETR (estadificacin local)
antiandrgenos, previamente a la introduccin de inhibidor de Gammagrafa osea (metstasis seas)
la LHRH. PSA (muy sensible, poco especfico). Descarta cncer
Antiandrgenos (bicalutamida, flutamida, acetato de cipro prosttico, pero no diagnostica HPB
terona). Compiten con el receptor andrognico. Suelen utilizar FAP (muy especfica, poco sensible)
se con agonistas de la LHRH. El acetato de ciproterona, adems Biopsia (confirmacin)
de actuar como antiandrgeno, tiene un efecto progestgeno, Tratamiento Fitoterapia Localizado: prostatectoma
por lo que acta a nivel central, disminuyendo los pulsos de LH. Frmacos: finasterida, radical ms linfadenectoma
a-bloqueantes bilateral, radioterapia
Quimioterapia. No es muy efectiva en el adenocarcinoma prosttico. Ciruga: Avanzado: castracin:
Se han realizado tratamientos con frmacos, que son una mezcla de un adenomectoma: quirrgica (eleccin),
estrgeno y una mostaza nitrogenada (fosfato de estramustina), pero endoscpica farmacolgica
los estudios son contradictorios y las respuestas pobres. Existen estu o abierta
dios prometedores con el empleo de docetaxel en pacientes con tumo Tabla 9. Tabla-resumen de las caractersticas de la HPB y del adenocarcinoma
res hormonorresistentes (MIR 04-05, 1 36). prosttico

11
Manual CTO de Medicina y Ciruga, 9. edicin

Tras la realizacin de radioterapia como tratamiento de cncer de prstata medular mediante laminectoma quirrgica o radioterapia. Se pueden dis
localizado, los descensos de PSA van siendo paulatinos (a diferencia de la minuir los niveles de andrgenos mediante castracin quirrgica urgente,
prostatectoma radical) hasta conseguir un valor nadir, que es el valor m ketoconazol en altas dosis (recientemente eliminado de las guas clnicas) 0
nimo alcanzado tras el tratamiento y que se considerar referencia para el dietilestilbestrol intravenoso.
seguimiento posterior. Existen diferentes criterios para considerar el diag
nstico de recidiva bioqumica tras tratamiento con radioterapia: cuando se
constata la existencia de tres elevaciones sucesivas a partir del valor nadir, ../ MIR 1 3-14, 1 24, 1 27
cuando se evidencian niveles nadir+2 (criterio de la American SocietyofC/in i ../ MIR 1 2-13, 124
cal Oncology, ASCO), o con valores nadir+3 (criterio de Philadelphia). ../ MIR l l -12, 1 1 0
../ MIR 1 0-1 1 , 1 00
../ MIR 09-1 O, 98
Tratamiento de u rgencia ../ MIR 08-09, 1 06
../ MIR 06-07, 1 03, 233
La compresin medular por el cncer prosttico no tratado puede ser l a for ../ MIR 05-06, 1 06
ma de presentacin y constituye una urgencia importante. El objetivo del ../ MIR 04-05, 1 06, 136, 225
../ MIR 03-04, 91
tratamiento debe ser la supresin andrognica rpida o la descompresin

" El PSA elevado no es diagnstico de cncer de prstata. Puede co


I d e a s c l a v e 16 rresponder a una HPB. El diagnstico definitivo de cncer prostti
co precisa una biopsia.
" La HPB suele afectar a la zona periuretral de la glndula. El cncer
aparece en la zona perifrica. " Las metstasis lumbares son tpicas del cncer de prstata, pudien
do producir compresin medular.
" La HPB no guarda relacin con el cncer.
" El tacto rectal revela una prstata ptrea e irregular en el cncer de
" Tanto la HPB como el cncer tienen relacin con las hormonas prstata. Sin embargo, al principio puede no ser palpable ni visible
sexuales, y suelen aparecer en varones ancianos. en la ecografa (Tl ).

" El tratamiento mdico de la HPB consiste en a-bloqueantes (re " La principal complicacin quirrgica del cncer de prstata es la
lajan la musculatura uretral y del cuello vesical). inhibidores de la impotencia.
5a-reductasa (disminuye el tamao glandular) y fitoterapia. Esta l
tima no ha demostrado utilidad con parmetros objetivos. " Ante un sndrome de compresin medular por cncer de prstata,
nunca se deben emplear anlogos de la LHRH nicamente. Siempre
" El tratamiento definitivo de la HPB es la ciruga, que puede consistir deben asociarse antiandrgenos.
en reseccin transuretral o en ciruga abierta, dependiendo del ta
mao prosttico. " En el cncer de prstata, la indicacin ms clara de prostatectoma
radical es el estadio T2a.
" El cncer de prstata es casi siempre un adenocarcinoma, con gran
frecuencia multifocal. " El tratamiento fundamental del cncer de prstata diseminado es
la hormonoterapia.
" El cncer de prstata cada vez se diagnostica con ms frecuencia
en fase asintomtica. Cuando presenta clnica, puede consistir en
sntomas urinarios similares a la HPB.

Hombre de 77 aos que refiere clnica de prostatismo de aos de


C a s o s c l n i c o s /1 evolucin, y que presenta elevacin del PSA (89 ng/ml) y dolor en
columna lumbar desde hace 2 meses. Al tacto rectal, la prstata est
Paciente de 66 aos, intervenido de prostatectoma radical, hace 3 aumentada de tamao, de consistencia dura en ambos lbulos, su
aos por adenocarcinoma de prstata Gleason 8 (pT2b NOMO). Pre perficie nodular y lmites mal definidos. Tras realizarle una ecografa
senta, en el momento actual, una cifra de PSA srico de 1 2 ng/ml. transrectal con biopsias prostticas ecodirigidas, es diagnosticado
Seale cul de las siguientes afirmaciones le parece correcta: de un adenocarcinoma de prstata pobremente diferenciado, que
afecta a ambos lbulos y que infiltra las vesculas seminales. La
1) La supervivencia media en el momento actual es menor de 1 ao. gammagrafa sea confirma la presencia de metstasis en columna
2) La cifra de PSA est en rango normal, ya que existen otras fuentes lumbar. Qu tratamiento, de los siguientes, aconsejara en primer
de produccin del mismo. lugar?
3) El paciente puede tener una recidiva local o bien metstasis a dis
tancia. 1) Prostatectoma radical.
4) La utilizacin de bloqueo hormonal en este caso no es una opcin 2) Quimioterapia intensiva.
de tratamiento posible. 3) Hormonoterapia.
5) En caso de tratarse de una recidiva local, estara indicado realizar 4) Radioterapia pelviana externa.
ciruga de rescate para extirpar dicha masa. S) Braquiterapia prosttica.

RC: 3; MIR 05-06, 106 RC: 3; MIR 03-04, 91

0 5 H i p e r p l a s i a b e n i g n a de p r s t ata
y carcinoma prosttico
11
Urologa I O5
Un paciente de 67 aos acude a Urgencias por presentar en los l posmiccional, sensacin de tenesmo y nicturia de tres veces. Pre
timos das debilidad progresiva de miembros inferiores, dificultad senta cultivos negativos y PSA de 2, 1 . En la ecografa abdominal se
miccional e incontinencia fecal. En la exploracin fsica destaca cier objetiva una glndula prosttica de 43 cm 3 Al tacto rectal no se pal
ta hipotona anal, con una prstata muy sugerente de malignidad pan ndulos sospechosos. En el IPSS obtiene una puntuacin que
al tacto, y debilidad de extremidades, conservando la sensibilidad permite clasificar su sintomatologa de moderada-grave. Su actitud
tctil. Con el probable diagnstico de carcinoma de prstata metas deber ser:
tsico, cul de las siguientes opciones considera MENOS indicada
para el tratamiento de urgencia? 1 ) Debido a su edad, el primer paso ser iniciar tratamiento con fito
terapia.
1) Estrgenos intravenosos. 2) Debido a la gravedad de los sntomas se debe plantear ciruga de
2) Anlogos LHRH. entrada.
3) Ketoconazol (altas dosis). 3) Se debe iniciar tratamiento con a-bloqueantes.
4) Radioterapia. 4) La mejor opcin ser iniciar tratamiento combinado con
5) Orquiectoma bilateral. u-bloqueantes + inhibidores de la 5o.-reductasa.
5) Se debe iniciar tratamiento con inhibidores de la 5o.-reductasa.
RC: 2
RC: 4
A la consulta acude un paciente de 54 aos con molestias a la mic
cin. Refiere disminucin del chorro, dificultad para el inicio, goteo
Urologa

CARCINOMAS DEL TRACTO URINARIO

O R I E NTACIN Las preguntas sobre este tema suelen ser sencillas y repetitivas, aunque ltimamente ha aparecido como nuevo

M I R concepto el carcinoma in situ. El estudio del Desglose es especialmente importante, pero se debe tener
en cuenta que la tendencia parece orientada a preguntarse cada vez ms. Es fundamental la parte de tratamiento.

6.1. Carcinoma urotelial negra. Su edad de mxima incidencia se sita entre los 60-70 aos. De ellos,
el 90% son carcinomas transicionales, el 8% escamosos y el resto adenocar
cinomas.
El epitelio urotelial recubre el tracto urinario desde las papilas caliciales hasta
la uretra prosttica, ambas inclusive. En cualquiera de estos niveles pueden El adenocarcinoma primario vesical es un tumor raro, aunque es el que se ha
desarrollarse los tumores uroteliales, correspondiendo la mayora a la vejiga visto asociado a la extrofia vesical con mayor frecuencia.
(ms del 90%) y, ms raramente, al tracto urinario superior (5%) o la uretra (1 %).
La infestacin por Schistosoma haematobium aumenta la incidencia de car
Entre los factores etiolgicos (Tabla 1 O), se implican las aminas aromticas, cinoma escamoso vesical, as como la presencia de infeccin crnica o cat
presentes en las industrias textiles, qumicas y del caucho. El humo del tabaco ter vesical permanente.
es el principal factor de riesgo (50-60% aparecen en fumadores), aumentan
do el riesgo a mayor consumo.Tambin parece que podran jugar un papel
importante los edulcorantes artificiales (sacarina, ciclamato), la ciclofosfamida, RECUERDA
los acetiladores lentos (mayor riesgo) y muchas otras posibles etiologas.
No hay que confundir Schistosoma haematobium con Schistoso
ma mansoni, que produce hipertensin portal.
6.2. Carcinoma vesical
Histologa e historia natural
El carcinoma vesical es la segunda neoplasia urolgica en frecuencia. Apa Haciendo referencia al carcinoma de clulas transicionales, hay que di
rece ms habitualmente en varones (2-3:1 ) y ms en poblacin blanca que ferenciar tres formas de la enfermedad con comportamiento, prons-
tico y tratamiento completamente distintos
Carcinomas Factores etiolgicos (Tabla 1 1 y Figura 21 ). El 70% de los tumores
vesicales se presentan como tumores papilares
Transicionales (90%) Aminas aromticas (2-naftilamina): tabaco, industria textil,
(mejor pronstico) industria del caucho, colorantes de crecimiento principalmente endocavitario y
Fenacetinas crnicas frente de invasin nico. Un 1 0% son slidos,
Sacarina, ciclamato con invasin tentacular en profundidad y ex
tensin linftica y vascular temprana. El 20%
Ciclofosfamida (acrolena)
restante son formas mixtas. El primer grupo
Tabaco: ortofenoles, triptfano
suele corresponder a tumores superficiales de
Escamosos (8%) Schistosoma haematobium bajo grado histolgico, mientras que los sli
Litiasis, infecciones, catteres dos, con mayor frecuencia, son tumores infil
trantes de grado histolgico ms elevado. La
Adenocarcinomas (2%) Cistitis glandular
principal caracterstica de los tumores papila
Extrolia vesical res superficiales es la recurrencia, que ocurre
Tabla 1 O. Factores etiolgicos de los carcinomas del tracto urinario en un 50-75%, segn el grado y estadio. El 25%

- ------- - - --
11 -----
f
Urologa I O6
recurrirn y progresarn en grado y estadio, y nicamente el 1 5% aca El tercer grupo que merece mencin aparte es el carcinoma in situ. A pesar
bar desarrollando un tumor infiltrante o metastsico. La mayora de los de encontrarse limitado al urotelio, por lo que es superficial, est formado
tumores infiltrantes se encuentran confinados a la vejiga en el momen por clulas poco diferenciadas con displasia grave. Tiene una alta tasa de
to del diagnstico, y slo un 20-25% presentan extensin ganglionar recidiva y progresa hacia tumor infiltrante en el 50-75% de los casos. Este
O metastsica. El 50% desarrollarn metstasis a distancia, a pesar del mal pronstico le confiere un carcter completamente distinto del carcino
tratamiento (MIR 04-05, 259). ma in situ de otras regiones, en las que se considera el estadio inicial de la
enfermedad tumoral. El carcinoma in situ puede estar asociado a focos de
carcinoma superficial (26%) o infiltrante (60%) o bien encontrarse de forma
aislada, siendo generalmente multifocal tanto en vejiga como en otros pun
T: define el tumor tos del urotelio (MIR 07-08, 1 02).
Tis: carcinoma in situ (plano) Se pueden encontrar en el mbito vesical distintas lesiones benignas que no
Ta: carcinoma papilar no infiltrante se asocian con el desarrollo de cncer: los nidos de Von Brunn, la cistitis qus
Tl: tumor que invade tejido conjuntivo subepitelial tica y glandular originados en procesos inflamatorios o irritativos crnicos,
T2: tumor que invade msculo: y que probablemente sean distintas manifestaciones de un mismo proceso,
T2a: tumor que invade la mitad interna aunque pueden plantear el diagnstico diferencial (ocasionalmente se han
- T2b: tumor que invade la mitad externa descrito adenocarcinomas vesicales asociados a la cistitis glandular). Otras
lesiones benignas seran el adenoma nefrognico, el plipo simple, el papi
T3: tumor que invade tejido perivesical: loma invertido o el papiloma velloso.
- T3a: microscpicamente
- T3b: macroscpicamente (masa extravesical) Diagnstico
T4a: tumor que invade prstata, tero o vagina La hematuria macroscpica o microscpica monosintomtica es el hallaz
T4b: tumor que invade pared plvica o pared abdominal go ms frecuente, presente en el 75% de los pacientes (MIR 1 2-13, 1 26;
N: define la afectacin ganglionar MIR 06-07, 96). La presencia de microhematuria asintomtica, descubierta
durante estudios de cribado, slo se relaciona con enfermedad significati
Nx: metstasis ganglionar regional desconocida
va en menos del 2% de los casos. Pueden encontrarse sntomas irritativos
NO: ausencia de metstasis ganglionar regional (escozor, polaquiuria, tenesmo) en el 25-30%, solos o acompaando a la he
Nl: metstasis a un solo ganglio entre 2 y 5 cm maturia. La presencia de un sndrome cisttico no justificado por infeccin
N2: metstasis en un ganglio mayor de 5 cm o mltiples no mayores o litiasis debe hacer sospechar la presencia de un carcinoma vesical, espe
de S cm
cialmente por su asociacin con el carcinoma in situ. Con menor frecuencia,
N3: metstasis mayores de 5 cm
el paciente consulta por dolor en flanco por obstruccin ureteral, plvico o
M: define las metstasis por edema en miembros inferiores (extensin linftica). La exploracin fsica
MO: no metstasis suele ser irrelevante, salvo en la enfermedad avanzada.
M 1: metstasis a distancia
Las citologas urinarias suponen una prueba sencilla y fiable que debe ser
Tabla 1 1 . Estadificacin del carcinoma vesical realizada en todos los casos de hematuria asintomtica o sospecha de tu
mor vesical (MIR 1 0- 1 1 , 108). Su sensibilidad depende del grado de dife
renciacin del tumor vesical, alcanzando el 75-1 00% en tumores de alto
grado y carcinoma in situ, siendo en este ltimo un mtodo diagnstico
ms rentable que la ecografa, la TC, la urografa o la biopsia mltiple. Son
especialmente tiles en el seguimiento de pacientes sometidos a reseccin
transuretral en combinacin con la cistoscopia.

Entre las pruebas radiolgicas destacan la ecografa (con una sensibilidad


del 80%, pero poco til para el diagnstico de las neoplasias del tracto uri
nario superior [Figura 22]) y la UIV (capaz de detectar la presencia de tumor
en el 60% de los casos).

En la UIV, adems de la presencia de defectos de replecin, se puede sugerir


el diagnstico de tumor vesical, rigidez y falta de distensibilidad vesical, la obs
truccin de un urter o el desplazamiento de la vejiga, entre otros. En caso de
dudas sobre el tracto urinario superior, se recurrir a la pielografa retrgrada
en el momento de realizar la cistoscopia. sta es fundamental para la evalua
cin del tumor vesical; puede realizarse bajo anestesia local cuando existan
1 . Mucosa ----+ Ta, Tis 4. Grasa - -
- -- T3
dudas con las pruebas realizadas previamente, pero si el diagnstico de pre
2. Submucosa Tl 5. rganos vecinos - T4
suncin es firme, y dado que en todo tumor debe realizarse reseccin transu
3. Muscular T2
retral para evaluar el grado de infiltracin, se puede esperar a tener al paciente
en quirfano bajo anestesia general o raqudea para practicarla. En un 10%
Figura 21. Esquema de la estadificacin del tumor vesical de los casos de cistoscopia con citologas positivas no se encuentra tumor en

11
Manual CTO de Medicina y Ciruga, 9. edicin

la vejiga, lo que puede deberse a la presencia de carcinoma in situ, tumor en radioterapia, que nicamente ha demostrado incrementar el tiempo libre
vas urinarias altas, carcinoma ductal de prstata o falso positivo de la prueba de recidiva local, sin aumentar la supervivencia. La quimioterapia combina
(generalmente, por inflamacin de la pared vesical o por tratamiento conco da con M-VAC (metotrexato, vinblastina, adriamicina, cisplatino) se reserva
mitante con radioterapia o quimioterapia endovesical). para pacientes en los que existe afectacin ganglionar o metastsica, con
respuestas completas en el 1 5-30% y parciales en el 30-40%. Tras la cistecto
ma, los urteres son derivados generalmente a segmentos intestinales o a
piel, pudiendo realizarse estomas no continentes o reservorios continentes
directamente al remanente uretral.

RECUERDA
La adriamicina (doxorrubicina) es un quimioterpico que puede
producir cardiotoxicidad.

6.3. Tumores
del tracto urinario superior

Entre el 2-10% de los tumores uroteliales se encuentran localizados entre los


clices y los orificios ureterales. En su etiologa estn implicados los mismos
factores que para el carcinoma vesical, a los que habra que aadir la nefropata
por abuso de analgsicos (fenacetinas) y la nefropata de los Balcanes. En el 70-
80% de los pacientes aparece hematuria macroscpica, siendo el dolor clico
por obstruccin ureteral la segunda queja en frecuencia de aparicin.
Figura 22. Ecografa. Tumor vesical con zonas calcificadas

La TC se utiliza para la estadificacin de la enfermedad infiltrante, ya que RECUERDA


aporta informacin limitada sobre la infiltracin tumoral. La RM consigue
La hematuria tambin es la manifestacin ms frecuente del hiper
mejores imgenes de la cpula vesical por sus cortes sagitales, pero no nefroma.
aporta mayor informacin que la TC.

Tratamiento
Diagnstico
Todo el proceso diagnstico va encaminado a establecer si el tumor vesical
es superficial o infiltrante, ya que el tratamiento vara radicalmente en fun La sistemtica diagnstica es bsicamente la misma que para el tumor vesi
cin de este hecho. cal. Es decir, ecografa (detectar hidronefrosis del lado del tumor), UIV (se
ver un defecto de replecin no compatible con un clculo o una anulacin
Los tumores superficiales son manejados mediante reseccin transuretral. funcional de ese sistema excretor, aunque hoy ha sido desplazada por URO
Dada la alta frecuencia de recidivas, la mayora se tratan posteriormente TAC con contraste) y TC para la estadificacin (Figura 23).
con instilaciones endovesicales (quimioterapia o inmunoterapia local) que
disminuyan la aparicin de nuevos tumores. Entre los quimioterpicos em En estos tumores, la citologa urinaria aumenta su eficacia si se obtiene de
pleados se encuentran la mitomicina, la tiotepa, la adriamicina o la epirru forma selectiva, cateterizando el urter del lado afectado. Otros elementos
bicina. Con todos ellos se consigue reducir las recidivas alrededor de un de diagnstico son las biopsias por cepillado y la ureteropieloscopia, ele
20%. La inmunoterapia endovesical con BCG (bacilo Calmette-Guerin) es, mento que, adems de diagnstico, puede tener un fin teraputico en tu
sin duda, la ms eficaz, disminuyendo el porcentaje de recidivas en un 40%, mores de pequeo tamao y aspecto superficial (Figura 24).
siendo tambin tratamiento de eleccin en el carcinoma in situ, con una efi
cacia del 70% (MIR 11-12, 108; MIR 05-06, 103). Sin embargo, no se utiliza
en los tumores vesicales como primera lnea, reservndose para los tumores Tratamiento
de riesgo o con mltiples recidivas, ya que tambin se acompaa de mayor
nmero de efectos secundarios y de complicaciones, entre los que se en El tratamiento estndar es la nefroureterectoma radical con extirpacin de
cuentran cistitis febril, sndrome pseudogripal, y las ms graves de sepsis, un rodete perimetico vesical; tal extensin es necesaria por la tendencia a
prostatitis granulomatosa, neumonitis e incluso muerte. Estos cuadros pre la recidiva de estos tumores.Es planteable, sin embargo, el empleo de trata
cisan tratamiento tuberculosttico completo al menos de 3-6 meses. mientos conservadores en caso de tumores papilares, no infiltrantes, nicos
y de pequeo tamao.
En la enfermedad invasora o infiltrante, tras la reseccin transuretral para
evaluar la afectacin parietal, el tratamiento de eleccin es la cistectoma Es necesario un seguimiento posterior de la vejiga y del rin contralateral,
radical. La quimioterapia sistmica, adyuvante o neoadyuvante, no parece ya que un 20-30% desarrollarn un tumor vesical metacrnico y un 2% en el
mejorar los resultados de la ciruga aislada. Igualmente se puede decir de la sistema colector contralateral.

06 C a r c i n o m a s d e l t r a c t o u r i n a r i o
-11
Urologa I O6

!
Fumador+ Hematuria

Sospecha de tumor urotelial

t ECOrvp
Citologas + Pruebas de imagen <
UIV

para'"T,,,
No concluyente
'"T
Concluyente
para ,kal

e+ +,+
Cistoscopia El) ---------- RTUV ------

Biopsia vesical El) ------Tis ;,, T2 TA, T1

e+
Sospechar tumor BCG Cistectoma QuimoBCG (G3)
Tracto urinario superior
Ureterorrenoscopia Revisiones
Pielografa retrgrada Cistoscopia Reciiva TisLctoma Rediva
Citologas selectivas + Citologa
Cepillado ureteral

Figura 24. Algoritmo diagnstico-teraputico en el tumor de vejiga


(MIR 03-04, 82)

./ MIR 1 2-1 3, 1 26
./ MIR 1 1 -1 2, 1 08
./ MIR 10-1 1 , 1 08
./ MIR 07-08, 102
./ MIR 06-07, 96
./ MIR 05-06, 103
Figura 23. Tumor vesical con zonas calcificadas. (A) UIV con defecto de ./ MIR 04-05, 259
replecin en pelvis renal derecha; (B) TC en fase excretora con tumoracin ./ MIR 03-04, 82
pilica derecha

" Cuando se trata de un carcinoma in situ: sntomas irritativos (pola


Ideas cl ave quiuria, disuria, tenesmo . . . ).

" El carcinoma vesical ms frecuente es el urotelial, siendo el tabaco " El mejor mtodo para la estadificacin local es la reseccin transu
el principal factor de riesgo. retral.

" El carcinoma escamoso se relaciona con la esquistosomiasis (S. hae " Prueba ms sensible para el diagnstico de carcinoma in situ: cito
matobium). loga urinaria.

" El adenocarcinoma vesical se relaciona con el antecedente de ex " Conducta ante un carcinoma in situ: tratar con bacilo Calmette
trofia vesical. Guerin (BCG) y revisiones (cistoscopia y citologas).

" El carcinoma papilar superficial y el carcinoma in situ son muy re " Actitud ante un tumor superficial: reseccin transuretral. Posterior
currentes. mente, revisiones (cistoscopia y citologas).

" Clnica ms frecuente del carcinoma urotelial: hematuria, ms tpico " Actuacin ante un tumor infiltrante (afectacin capa muscular): cis
con cogulos. tectoma.
Manual CTO de Medicina y Ciruga, 9. edicin

antibitico y le ha realizado cultivos que han sido negativos. Se le


C asos clnicos . realiza una ecografa y una flujometra que son normales y una uro
grafa intravenosa que no evidencia alteraciones. En la cistoscopia
Varn de 57 aos, fumador, que consulta por hematuria terminal, no se observan lesiones intravesicales. Las citologas urinarias son
polaquiuria, urgencia y dolor miccional. Presenta citologas urina sugestivas de malignidad. Seale la INCORRECTA:
rias positivas de carcinoma urotelial, y el estudio anatomopatol
gico tras la reseccin transuretral es de carcinoma in situ difuso, 1) El tratamiento de eleccin ser la i nmunomodulacin vesical.
con intensa inflamacin crnica. El tratamiento estndar ser: 2) Es imprescindible la realizacin de biopsias para confirmacin del
diagnstico.
1) Instalacin del bacilo de Calmette-Guerin. 3) Si recidiva tras las instilaciones, se deben repetir los ciclos dos veces
2) Cistectoma radical. ms.
3) Instilaciones con mitomicina. 4) La cistectoma es el tratamiento final en muchos de estos pacientes.
4) Quimioterapia con cisplatino. 5) A priori no se encontrar infiltracin de la capa muscular.
5) Antiinflamatorios no esteroideos ms quinolonas durante 6 meses.
RC: 3
RC: 1
Una paciente de 58 aos acude por polaquiuria de 2 aos de evo
Mujer de 63 aos que es diagnosticada de carcinoma de clulas es lucin, nicturia y dolor hipogstrico. Niega haber visto sangre en la
camosas del trgono vesical, con invasin de la capa muscular. Cul orina y es fumadora. Seale la respuesta correcta:
sera su actitud teraputica en este caso?
1) Se deben realizar biopsias a esta paciente.
1) Radioterapia externa con 7.000 rads. 2) Una cistoscopia permitir confirmar el diagnstico.
2) Quimioterapia adyuvante, seguida de cistectomia radical. 3) Los tratamientos empleados hasta el momento han demostrado
3) Reseccin transuretral, seguida de inmunoterapia intravesical (BCG). una gran eficacia.
4) Cistectomia radical con extirpacin de cara anterior de vagina. 4) Muy probablemente los cultivos sern positivos.
5) Radioterapia externa, seguida de quimioterapia con cisplatino. 5) En todos los casos se detectarn > 1 O hemates/campo en el sedi
mento de orina.
RC: 4; MIR 03-04, 82
RC: 1
Un paciente de 64 aos acude a consulta por sndrome miccional de
2 meses de evolucin. Refiere que su MAP le ha dado tratamiento

06 . C a r c i n o m a s d e l t r a cto u r i n a r i o

11
1111

Urologa

TUMORES T ESTICULARES

ORI E N TAC I N Este tema es sencillo y rentable siempre que se seleccione lo realmente importante: la clnica y el diagnstico.

M I R Es fundamental la revisin profunda de los Desgloses. En cuanto al tratamiento, dado que no existe un protocolo
universalmente aceptado, es mejor conocer ideas generales.

7.1 . Etiol oga y epidemiologa los factores txicos, la exposicin a radiaciones, fuentes de calor, productos
para teido del cuero y estrgeno intrauterino durante el primer trimestre
del embarazo. Aunque entre un 8-25% de los pacientes presentan historia
Los tumores testiculares representan el 1 -2% de las neoplasias en varones, de traumatismo testicular, todos los autores parecen estar de acuerdo en
y son las neoplasias slidas ms frecuentes entre los 20-35 aos. Presentan que ste supone ms bien el motivo por el que se descubre una masa es
mayor incidencia a mayor desarrollo de la sociedad, y tambin en la raza crotal, y no su origen.
blanca ms que en las dems. La tasa de curacin es superior al 90%.

El 95% de ellos procede de clulas germinales y, aunque globalmente el 7.2. Anatoma patolgica
seminoma es el ms frecuente, la incidencia vara segn el grupo de edad
considerado. El 5% restante se reparte entre tumores del estroma gonadal
(1 -2%), linfomas (1%), gonadoblastomas (clulas germinales y del estroma), Los tumores testiculares se suelen originar como una masa testicular intrapa
metstasis y otros. renquimatosa.A partir de ah, pueden quedarse localizados o metastatizar. La
localizacin ms frecuente de metstasis son los ganglios retroperitoneales
Los factores de riesgo para el desarrollo de tumor testicular son teste hi (casi siempre es el primer escaln en la diseminacin); posteriormente, puede
poatrfico, disgenesias gonadales, historia familiar de neoplasias testicula aparecer la afeccin mediastnica, de vsceras abdominales y de pulmn.
res, tumor previo en el otro teste, exposicin a estrgenos y criptorquidia.
Los testculos no descendidos tienen mayor riesgo de desarrollar tumores, Hay que tener en cuenta que el testculo izquierdo drena a los ganglios pa
y esta probabilidad aumenta si la situacin del teste es intraabdominal. raarticos y prearticos a nivel de L2.El derecho drena a ganglios interaorto
Asimismo, el testculo contralateral, aunque de localizacin escrotal, tiene cavos, precavas y prearticos, y tambin a nivel del hilio renal. La disemina
una incidencia mayor; de hecho, el 20% de los tumores en pacientes con cin hematgena es menos frecuente, salvo en el coriocarcinoma, va vasos
criptorquidia se desarrollan en el testculo no criptorqudico. Por estas ra espermticos, siendo los puntos ms habituales de metstasis: pulmn, h
zones, los testculos no descendidos deben descenderse, preferiblemente gado, hueso y sistema nervioso central, por orden de frecuencia.
entre el primer y el segundo ao de edad, para facilitar su seguimiento y,
con ello, la deteccin de un eventual tumor (MIR 09-1 o, 1 74). Los tumores testiculares se dividen en dos grupos, tumores que no derivan
de las clulas germinales (5%) y tumores que derivan de las clulas germi
Una vez alcanzada la pubertad, un teste criptorqudico no descendido pro nales (95%); a su vez, estos ltimos se dividen en seminomatosos y no semi
bablemente deba extirparse, dado que estos testculos pierden la capacidad nomatosos (Tabla 1 2):
de espermatognesis y conservan su potencial malignizante. No obstante, Seminoma. Puede aumentar el tamao testicular hasta 1 O veces sin
hay autores que prefieren no extirparlos, siempre y cuando se puedan des distorsionar su morfologa. Ocasionalmente se presenta extratesticular
cender a la bolsa escrotal, ya que an pueden mantener su funcin endocri en mediastino (la ms frecuente), retroperitoneo, o regin pineal de for
na (secretora de testosterona). ma primaria. Se han descrito tres tipos histolgicos: seminoma tpico,
anaplsico y espermatoctico (ms frecuente en individuos mayores de
Otros factores que se pueden encontrar relacionados son las hernias ingui 50 aos). El seminoma puro no es productor de marcadores tumorales,
nales infantiles (no demostrado) y la orquitis urliana secundaria al paramixo pero hasta en un 1 5% de los casos pueden aparecer clulas del sincitio
virus causante de la parotiditis (siempre que haya producido atrofia); y, entre trofoblasto, produciendo elevaciones de la -HCG.

-11-
Manual CTO de Medicina y Ciruga, 9. edicin

Carcinoma embrionario. Tiende a metastatizar de forma temprana. 7 .3. Clnica


Forma masas ms irregulares y heterogneas que otros tumores.
Tumor del seno endodrmico. Este tumor fue descrito inicialmente
como una forma rara de tumor infantil. Se encuentran elementos de La manifestacin ms frecuente es como masa escrotal indolora. Con mu
saco vitelino en el 38% de los tumores testiculares del adulto. cha menor frecuencia, el motivo de consulta est originado por la presen
Coriocarcinoma. En el estudio histolgico tiene que contener elemen cia de metstasis ganglionares (masas supraclaviculares o abdominales
tos de sincitiotrofoblasto y citotrofoblasto para ser considerado como (MIR 09-1 O, 102), o efectos endocrinos (ginecomastia, pubertad precoz).
tal. En el momento del diagnstico, generalmente existen metstasis El 1 0% de los tumores se presenta como escroto agudo en la urgencia.
a distancia (suelen ser va hematgena [MIR 1 3-14, 1 25]) y un tumor
primario testicular pequeo. Raramente es puro (MIR 06-07, 94).
Teratoma. Por definicin, se encuentra formado al menos por dos capas RECUERDA
distintas de clulas germinales (endodermo, mesodermo o ectodermo).
Existen algunas enfermedades, como la sarcoidosis, que aumentan
Teratocarcinoma. Tumor mixto con reas de teratoma y de carcinoma el tamao testicular sin existir un tumor.
embrionario. Un 64% tiene tambin reas de seminoma.
Gonadoblastoma. Contiene grandes clulas similares al seminoma y otras
menores, como clulas de Sertoli inmaduras o de la granulosa. Se asocia
preferentemente a las gnadas disgenticas y estados intersexuales. 7 .4. Diagnstico
Tu mores de clulas de Sertoli. Forman reas tubulares similares a los
tbulos del testculo prepuberal normal. De comportamiento benigno,
es excepcional que metastaticen. La exploracin fsica de una masa indolora, sin signos inflamatorios y de largo
periodo de evolucin, debe hacer sospechar un tumor testicular (Tabla 1 3).

RECUERDA
TNM
El tumor de clulas de Sertoli se ha asociado al sndrome de Peutz 1
Jeghers (vase Apartado de Sndromes de poliposis, en la Seccin pTis lntratubular
de Digestivo). pTl Testculo y epiddimo sin invasin vascular/linftica
pT2 Testculo y epiddimo con invasin vascular/linftica o tnica
vaginal
Tumores de clulas de Leydig. Pueden verse cristaloides en su cito pT3
pT4 Afectacin de cordn espermtico
plasma. Generalmente son benignos. Pueden ser hormonalmente acti Escroto
vos, produciendo pubertad precoz o feminizacin.
Linfoma. Tanto como localizacin secundaria o como origen primario Nl Ganglios menores de 2 cm
N2 Ganglios entre 2-5 cm
(menos frecuente), una masa testicular en un varn mayor de 50 aos N3
sugiere, en primer lugar, el diagnstico de linfoma. Tras la orquiectoma Ganglios mayores de 5 cm
o la biopsia testicular para el diagnstico de certeza, el tratamiento no Mla Metstasis en ganglios no regionales o pulmn
vara respecto a los linfomas habituales. Mlb Metstasis viscerales no pulmonares
Sx Marcadores tumorales sricos no disponibles
SO Niveles de marcadores normales
Tumores de clulas germinales Tumores del estroma Sl LDH < 1 ,5 x n ; y P-HCG < 5.000; y AFP < 1.000
Tumores de un tipo histolgico 52
LDH entre 1 ,5 x n y 1 O x n o P-HCG entre 5 0. 00 y 50.000
Seminoma: 53 o AFP entre 1 .000 y 10.000
Tpico LDH > 1 O x n o p-HCG > 50.000 o AFP > 10.000
Anaplsico
n = lmite superior normal de LDH
Espermatoctico
Agrupacin por estadios TMR testculo (abreviado)*
Carcinoma embrionario Estadio O pTis NO MO
Poliembrioma Tumores de clulas
Tumor del saco vitelino (seno endodrmico) de Leydig Estadio I pT14 NO MO
Coriocarcinoma Tumores de clulas Estadio IIA Cualquier pT Nl MO
Teratoma: de Sertoli
Tumores estructuras Estadio 118 Cualquier pT N2 MO
Maduro
gonadales primitivas Estadio IIC Cualquier pT N3 MO
- Inmaduro
- Con transformacin maligna Estadio 111 Cualquier pT Cualquier N Ml
Tumores de ms de un tipo histolgico
* No tiene en cuenta marcadores tumorales
Teratocarcinoma
Otros Tabla 1 3. Estadificacin clnica y patolgica
Tumores germinales + estromales
Un elemento importante, tanto para su diagnstico como para el seguimien
Gonadoblastoma to, son los marcadores tumorales: a.-fetoprotena (AFP) (MIR 07-08, 1 00) y
Tabla 1 2. Tumores de testculo. Clasificacin histolgica fraccin P de la gonadotropina corinica humana (P-HCG) (Tabla 14).

0 7 Tu m o r e s testiculares
Urologa I O7
Tumores germinales: marcadores tumorales genrico que se denomina "escroto agudo''. caracterizado por el aumento
Tumor con una palabra aumenta P -HCG: doloroso de volumen del contenido escrotal, acompaado o no de signos
- Coriocarcinoma inflamatorios (Tabla 1 5 y Figura 25):
- Seminoma (en la mujer, el equivalente es el disgerminoma) Orquiepididimitis. Suele presentarse con dolor intenso, enrojecimien
to cutneo, fiebre, y a veces sintomatologa miccional.En la exploracin,
Tumor con dos palabras aumenta a-fetoprotena: la elevacin del testculo (signo de Prehn) disminuye el dolor (signo de
Prehn positivo). En su etiologa se deben considerar grmenes de trans
- Carcinoma embrionario (MIR 08-09, 97)
misin sexual en pacientes adultos menores de 35 aos, y uropatge
- Tumor del seno endodrmico
nos (E. coli) si superan esta edad.
Tabla 14. Marcadores tumorales en tumores germinales (vlidos para ovario Torsin del cordn espermtico. Suele aparecer en la infancia o la
y testculo) adolescencia, con dolor de aparicin brusca y signos cutneos inflama
torios crecientes a medida que progresa el cuadro. El teste se encuentra
La AFP es sintetizada por clulas del saco vitelino y, por tanto, est presente horizontalizado, y ocasionalmente puede palparse la espiral del cordn
en tumores de saco vitelino o seno endodermal, y en los carcinomas em torsionado. En este caso, la elevacin del testculo incrementa la sensa
brionarios. El seminoma nunca produce AFP (MIR 04-05, 1 07). En cualquier cin dolorosa (signo de Prehn negativo).
caso, se debe tener en cuenta que la AFP es un marcador inespecfico, y se Hidrocele y espermatocele. Son dos cuadros que raramente se presentan
podra encontrar elevado en enfermedades hepticas benignas o malignas, de forma brusca y con dolor agudo, fcilmente diferencia bles por la explo
algunas neoplasias pancreticas y de la va biliar o en la ataxia-telangiectasia. racin y su transiluminacin positiva y, ante la duda, mediante ecografa.

RE C UERDA Orquitis Polaridad conservada


Prehn MEJORA el dolor
La AFP tambin se eleva en lquido amnitico en alteraciones del
cierre del tubo neural. Vase Seccin de Ginecologa y Obstetricia. Torsin del cordn Testculo horizontalizado
espermtico Prehn EMPEORA el dolor

La fraccin p de la HCG es producida por las clulas del sincitiotrofoblasto Masa palpable indolora
presentes en el coriocarcinoma y, tambin de forma ocasional y de forma Tabla 1 5 . Diagnstico diferencial de los tumores testiculares
aislada, en algunos seminomas.

En conjunto, el 70% de los tumores testiculares produce algn marcador, lue


go existe hasta un 30% de tumores con marcadores negativos al diagnstico.

La vida media de la AFP es de 7 das, frente a 3 das de la 13-HCG. Este dato es


importante a la hora de valorar la posible presencia de enfermedad residual,
ya que la elevacin persistente de uno de estos marcadores despus del
tratamiento supone la existencia de tumor no eliminado.

La ecografa testicular es un mtodo sencillo y fiable para la diferenciacin en


tre masas slidas y qusticas, y su localizacin exacta intratesticular o depen
diente de los anejos. Cuando, a pesar de los marcadores, los datos ecogrficos
son sugerentes de tumor, est indicada la exploracin quirrgica a travs de
una incisin inguinal, para evitar la posibilidad terica de implantes tumorales
en la piel escrotal y tener mejor control de pedculo vasculolinftico a nivel del
cordn espermtico. Si la exploracin confirma la presencia de una masa, el
testculo debe ser extirpado (orquiectoma radical) (MIR 05-06, 1 07).

La evaluacin de la extensin tumoral se completar mediante TC tora


coabdominal. As se constatar si la enfermedad est limitada al testculo
(estadio 1), o existe afectacin de ganglios infradiafragmticos (estadio 11),
o si hay incluso extensin supradiafragmtica o a rganos slidos (estadio
111). El sistema de estadificacin utiliza mltiples variaciones, pero quiz la Figura 25. Pieza macroscpica de tumor testicular
clasificacin ms aceptada sea la expuesta anteriormente en la Tabla 1 3.

7.5. Diagnstico diferencial 7.6. Tratamiento

Aunque frecuentemente la existencia de un tumor testicular no plantea du Hasta la fecha no existe un protocolo nico de tratamiento, pudiendo variar
das diagnsticas, existe una serie de patologas testiculares que, junto con incluso de un centro a otro. Siempre se realizar orquiectoma radical va in
los tumores, pueden presentarse en algn momento dentro del cuadro guinal. Posteriormente, el patlogo informar de la estirpe histopatolgica.
Manual CTO de Medicina y Ciruga, 9. edicin

A continuacin, el estudio de extensin mediante TC toracoabdominoplvi Estadio llc-111. Antes de la aparicin de la actual quimioterapia, la su
ca y nuevos marcadores postorquiectoma. En funcin de la histologa y del pervivencia era del 5-10%. Actualmente, el esquema de tratamiento
estadio, se asignar un tratamiento a cada paciente (MIR 1 2- 1 3, 128). mayoritariamente aceptado es quimioterapia primaria.

El tratamiento de la neoplasia testicular se resume en la Figura 26.


RECUERDA
La orquiectoma en el cncer de testculo es va inguinal. En el cn
cer de prstata, va escrotal. Tumor testicular
1

Seminoma Orquiectoma radical

Se caracteriza por su gran radiosensibilidad, de ah que la radioterapia haya Seminoma No seminoma


sido la base del tratamiento de estos tumores. Actualmente, la quimiotera
pia obtiene resultados similares.
t t
lla,llb la lla, llb
Estadio l. El tumor tericamente est limitado al testculo. No obstante,
se sabe que hasta un 20% de pacientes presentan micrometstasis a
Observacin
Si FR:
Observacin
Linfadenectoma
+++
invasin
t
Quimioterapiz
Radioterapia
ganglios retroperitoneales, que en el momento del diagnstico no se > 4 cm rete testis Quimioterapia (EEU U) vascular
Quimioterapia o
detectan. La presencia de micrometstasis se ha relacionado con dos (UE) linftica
factores de riesgo: tumor de ms de 4 cm e invasin tumoral de la rete Radioterapia
Quimioterapia
testis. En estos pacientes se aconseja administrar radioterapia o quimio
terapia (cisplatino). Con esto, frente a los pacientes que optaron nica Seminoma/ No seminoma
mente por observacin, la recidiva desciende del 20 al 5%. 11(-111

Quimioterapia
RECUERDA
Figura 26. Algoritmo de tratamiento de la neoplasia testicular
El cisplatino produce vmitos con mucha frecuencia. Otro efecto
secundario es su nefrotoxicidad.
Masas residuales
Estadio lla-llb. En este caso, el tumor ya est extendido a ganglios re Se define como masa residual la existencia de conglomerados adenopticos
troperitoneales y requiere, por tanto, tratamiento agresivo. Se dispone tras tratamiento quimioterpico o radioterpico. Cuando el tumor primario
de dos opciones: radioterapia sobre las cadenas afectadas (teniendo es un semi noma y existen masas residuales, la actuacin a seguir (Figura 27)
en cuenta el teste afectado, se irradiar a unas cadenas ganglionares u ser: si la masa residual es inferior a 3 cm, tiene muy pocas probabilidades de
otras), o quimioterapia BEP (cisplatino, etopsido y bleomicina). Ambas contener tumor residual y no requiere ms que observacin. Pero cuando es
obtienen resultados muy similares. superior a 3 cm, se debe realizar una PET (tomografa por emisin de posi
Estadio llc-111 (estadios avanzados). El tumor tiene metstasis gan trones}, si existiera esta posibilidad a nivel tcnico, pues detecta con una alta
glionares retroperitoneales superiores a 5 cm, o afectacin supradia sensibilidad y especificidad la presencia de tumor residual. Si no se dispone
fragmtica o de vsceras slidas. La radioterapia deja de ser una opcin de una PET o si sta es positiva, se realizar ciruga de la masa.
teraputica. La quimioterapia (BEP) es la nica posibilidad, presentando
una tasa de curacin de alrededor del 80%.

Tumores no seminomatosos Mfili@@N


Estadio l. Existen tres posibilidades teraputicas tras la orquiectoma:
Observacin y seguimiento peridico.Tasas de recadas de un 20%. Seminoma

No seminoma
Linfadenectoma retroperitoneal de estadificacin, pues as se tie
ne certeza del estadio y se reducen a un 10% las recidivas (en Euro t

l +
pa no est extendida esta prctica). < 3 cm > 3 cm

Quimioterapia profilctica (cisplatino), y as se reducen al 5% las re


cidivas. +/- PET

+
Observacin --- e / $
En aquellos casos en que exista invasin vascular en la pieza de orquiec
toma, la tasa de recidivas asciende al 50%. Parece lgico, en estos casos,
inclinarse de entrada por una de las dos ltimas opciones. La tasa de Exresis de la masa residual

curacin alcanza el 98%.


Estadio lla-llb. Histricamente se realizaba linfadenectoma retroperi
15% tumor viable
toneal completa como nico tratamiento, pero ante tasas de recidiva 50% necrosis 35% teratoma

no desdeables, actualmente se indica quimioterapia de inicio (BEP). La


tasa de supervivencia supera el 95%. Figura 27. Algoritmo de tratamiento de las masas residuales

0 7 Tu m o r e s t es t i c u l a r e s
11
Urologa I O7
Cuando el tumor primario es un tumor no seminomatoso, se debe realizar ./ MIR 1 3-14, 1 25
exresis de dicha masa siempre, con independencia del tamao. ./ MIR 1 2-13, 128
./ MIR 09-10, 1 02, 1 74
La histologa de estas masas residuales, una vez extirpadas. puede ser: te ./ MIR 08-09, 97
jido necrtico hasta en un 50% de las ocasiones, tumor viable en un 1 5% ./ MIR 07-08, 1 00
y teratoma en un 35% (stos, dejados a libre evolucin, pueden convertir ./ MIR 06-07, 94
./ MIR 05-06, 1 07
se en teratomas malignos o producir procesos compresivos con su creci
./ MIR 04-05, 1 07
miento).

I d e a s c l a v e .S
" El seminoma nunca produce a - fetoprotena.

" Como tratamiento, la orquiectoma por va inguinal se realiza en to


" El tumor testicular es la neoplasia slida ms frecuente en el varn dos los casos.
joven. La tasa de curacin es mayor del 90%.
" El semi noma es radiosensible. Por ello puede utilizarse radioterapia
" El tumor testicular ms frecuente es el semi noma. No obstante, esto como tratamiento en los primeros estadios. Si se trata de un estadio
es muy variable segn la edad del paciente. avanzado, se empleara quimioterapia.

" Una masa testicular por encima de los 50 aos sugiere linfoma. " Los tumores no seminomatosos se tratan con orquiectoma + qui
mioterapia. No obstante, si es u n tumor limitado al testculo, puede
" El tumor tpico de las disgenesias gonadales es el gonadoblastoma. plantearse la vigilancia tras la orquiectoma.

" La clnica ms frecuente es una masa escrotal indolora.

Casos c l nicos ., 1
Un hombre de 31 aos consulta por la presencia de una masa pal
pable en el teste derecho, de 1 mes de evolucin, no dolorosa. Su
urlogo le realiza una ecografa testicular en la que se evidencia una
Qu diagnstico, entre los siguientes, es el ms probable en un lesin hipoecoica, bien delimitada, intratesticular. Los marcadores
nio de 8 aos con signos inequvocos de pubertad precoz y que, en tumorales a-fetoprotena y 3-HCG son negativos. La actitud ms co
la exploracin, presenta una masa en el testculo derecho de 2 cm rrecta de entre las siguientes sera:
de dimetro?
1) Dado que los marcadores tumorales son negativos, se descarta
1) Tumor de clulas de Leydig. neoplasia testicular y requiere observacin.
2) Seminoma. 2) Repetir la ecografa testicular en un plazo de 3 meses.
3) Tumor del saco vitelino. 3) Realizacin de una TC toracoabdominoplvica.
4) Teratoma. 4) Biopsia transescrotal del testculo.
5) Coriocarcinoma. 5) Orquiectoma radical y esperar resultado del patlogo.

RC: 4 RC: 5; MIR 05-06, 1 07

11-
Urologa

TRAS PLANTE RENAL

Sospecha de no cumplimiento teraputico del protocolo inmunosupresor.


Lo ms importante de este tema son
Glomerulonefritis activa.
los tipos de rechazo, que se solapan
con Inmunologa. Se debe insistir en Expectativa de vida reducida por enfermedad de base del paciente.
el rechazo agudo, que conviene repasar Presencia de anticuerpos preformados frente a antgenos del donante
con las preguntas de aos anteriores. (MIR 04-05, 103).

En referencia a las contraindicaciones relativas del trasplante renal, se


debe decir que stas se han ido modificando a lo largo de los aos. al me
8.1. Indicaciones jorar la tcnica y los cuidados prequirrgicos y posquirrgicos. En muchas
ocasiones, el trasplante plantea menos riesgo que una hemodilisis crnica.
Actualmente, se consideran contraindicaciones relativas la edad avanzada,
Las dos enfermedades que ms comnmente abocan a una insuficiencia la oxalosis, la amiloidosis, la enfermedad iliofemoral oclusiva, las anomalas
renal terminal irreversible, tratable mediante un trasplante renal, son: del tracto urinario inferior o las alteraciones psiquitricas graves.
Glomerulonefritis.
Diabetes mellitus insulinodependiente.
8.3. Complicaciones
Otras causas importantes son:
Poliquistosis renal.
Nefroesclerosis hipertensiva. Las complicaciones que se pueden presentar son las siguientes:
Enfermedad de Alport. Rechazo. Puede ser hiperagudo, acelerado, agudo o crnico (Tabla 1 6).
Nefropata lgA. La clnica del rechazo agudo es fiebre, hipertensin y dolor en el rea del
Lupus eritematoso sistmico. injerto; produce oliguria, no poliuria (MIR 1 3- 1 4, 1 29; MIR 05-06, 1 01 ).
Nefroesclerosis. Recurrencia de la enfermedad en el rin trasplantado.
Nefritis intersticial. Complicaciones tcnicas. Complicaciones vasculares, hemorragia, hi
Pielonefritis. pertensin por estenosis de la arteria renal, trombosis venosa. compli
Uropata obstructiva. caciones del tracto urinario, necrosis tubular aguda, linfoceles.
Complicaciones no tcnicas. Infecciones bacterianas y oportunistas
Los mejores receptores son individuos jvenes cuyo fallo renal no se deba en relacin con la inmunosupresin, hiperglucemias. complicaciones
a una enfermedad sistmica que pueda daar el rin trasplantado o cau gastrointestinales, hiperparatiroidismo y tumores (cncer de piel y de
sar la muerte por causas extrarrenales. Generalmente se suele mantener labios, carcinoma in situ de crvix, linfomas no Hodgkin; guardan rela
al receptor en tratamiento con dilisis durante un cierto tiempo previo al cin con el tratamiento inmunosupresor). Puede aparecer hipertensin
trasplante. debida a enfermedad en los riones originales, como consecuencia de
rechazo, por estenosis de la anastomosis de la arteria renal o por toxici
dad renal por ciclosporina (MIR 09-1 o, 97).
8.2. Contraindicaciones
P R E G U N TA S ./ MIR 1 3-14, 129
Las contraindicaciones absolutas son las siguientes:
Infeccin activa.
M I R ./ MIR 09-1 O, 97
./ MIR 05-06, 101
Enfermedad maligna que no pueda ser erradicada. ./ MIR 04-05, 103
- Urologa I O8
Rechazo Inicio Patogenia Patologa asociada Tratamiento
H iperagudo Minutos. das Ac. preformados Trombosis microvascular Nefrectoma del injerto
CID Isquemia o infarto
Act. del complemento PMN en capilares
Dao endoletelial
Acelerado Das Celular (+Ac) Vasculitis necrotizante Bolos de esteroides
Respuesta 2." a Ag-HLA Ac monoclonales
Agudo Semanas Celular (+Ac) Forma vascular: mediada por Ac Bolos de esteroides (la vascular suele ser resistente)
Infiltrado de linfocitos Forma celular: tubulointersticial Ac monoclonales
Crnico Meses, aos Humoral y celular ntima arterial aumentada No hay; control de HTA
Atrofia tubular
Glomerulopata
Tabla 16. Rechazo en el trasplante renal

I d e a s c l a v e Ji!S
" La clnica caracterstica del rechazo agudo es: fiebre, hipertensin y
dolor en el rea del injerto (MIR 13-14, 129; MIR 05-06, 101 ).

" Las causas ms frecuentes de insuficiencia renal crnica son la dia " El rechazo agudo produce oliguria, no poliuria.
betes mellitus y las glomerulonefritis.

C a s o s c l nicos VI
oliguria y disminucin en la concentracin urinaria de sodio. El diag
nstico ms probable sera:

Un paciente de 35 aos, con insuficiencia renal crnica, secundaria a 1) Crisis hipertensiva.


pielonefritis crnica recibe un trasplante renal de cadver con el que 2) Infeccin respiratoria.
comparta dos identidades en A y B y una en DR. Recibe tratamiento 3) Pielonefritis aguda del injerto renal.
inmunosupresor con ciclosporina A y corticoides en dosis estndar. 4) Recidiva de su enfermedad renal.
En el posoperatorio inmediato se observa buena diuresis, y no es S) Rechazo agudo del injerto renal.
necesario el tratamiento sustitutivo con hemodilisis. En el 5.0 da
de evolucin, el paciente presenta fiebre de 38 C, TA de 180/11 O, RC: 5
Urologa

,
U ROPATIA OBSTRUCTIVA

Tiempo de evolucin. La aguda suele cursar con dolor (clico nefrti


O R I E N TA C I N Tema poco preguntado hasta la fecha.
co), siendo la crnica ms frecuentemente asintomtica.
M I R Se debe tener una idea general
y aprender las Ideas clave. Lugar de la obstruccin:
Tracto urinario inferior (uretra y vejiga). Cursa con retraso para
iniciar la miccin, disminucin de fuerza y del tamao del chorro,
goteo terminal, hematuria, escozor al orinar, orina turbia, retencin
9.1. Caractersticas aguda de orina o incontinencia paradjica ("miccin por rebosa
miento").
Tracto urinario superior (urter y rin). Estos pacientes presen
Detencin del flujo de orina en cualquier punto entre los clices renales tan dolor en el flanco (rin y urter proximal), dolor en flanco con
y el exterior (Tabla 1 7). irradiacin a genitales (urter medio) o sndrome miccional (urter
Su importancia reside en el desarrollo potencial de insuficiencia renal, terminal).
por lo que tiene importancia la obstruccin urinaria bilateral o la unila
teral sobre rin nico funcionante. Despus de resolverse una obstruccin, sobre todo si es crnica, pue
Una obstruccin de ms de 1 mes de duracin puede dar lugar a un de producirse una fase de poliuria. Esto se debe a que, a nivel tubular,
dao renal funcional y estructural permanente. cuando se ralentiza crnicamente el flujo urinario, se genera una insen
sibilidad a la ADH transitoria (diabetes inspida nefrognica), de ah la
poliuria.
RE C U ERDA
La uropata obstructiva puede producir glomerulonefritis focal
y segmentaria. 9.3. Diagnstico

El diagnstico de seguridad, la valoracin de la evolucin y el pronstico


9.2. Clnica son ecogrficos. Adems, son tiles la anamnesis y la exploracin fsica, la
radiologa simple (conveniente en litiasis radioopaca), UIV (confirma una po
sible anomala funcional y til en litiasis radiotransparente), cistografa, estu
La forma de presentacin depende de los siguientes factores: dio metablico (til en prevencin de recidivas), TC abdominal, ETR, biopsia
Etiologa de la obstruccin. Presenta la clnica propia de la enferme prosttica dirigida, pielografa retrgrada, nefrostografa, cistoscopia, flujo
dad de base. metra, cistomanometra y citologa urinaria (MIR 07-08, 1 05).

lntraluminal lntraparietal Extraparietal (compresin extrnseca) Extraparietal (disfuncin neuromuscular)


Litiasis Estenosis congnita Urter retrocavo Vejiga neurgena
Tumores (hipernefroma, Estenosis postinfecciosa Rin en herradura Vejiga automtica: lesin sobre Ll-L3
urotelioma) Estenosis postraumtica Fibrosis retroperitoneal Vejiga tona: lesin bajo L 1 -L3
Necrosis papilar Estenosis isqumica Tumores (prstata, vejiga, ginecolgicos) Disfuncin de la unin pieloureteral
Cogulos Hiperplasia prosttica Reflujo vesicoureteral
Ligadura iatrgena de urteres
Tabla 1 7. Clasificacin de la uropata obstructiva (MIR 1 1 -12, 105)

------------- - -- -
-
Urologa I O9
9.4. Tratamiento
Manejo de la uropata obstructiva

Es necesario restablecer el flujo urinario. La mayor parte de las veces se rea


liza mediante:

lnfravesical Supravesical

+
Litotricia. (globo vesical) (no globo vesical)
Correccin quirrgica.
t
Sonda vesical
Si la obstruccin es aguda y/o bilateral, la desobstruccin es urgente, y pue Ecografa renal

t
o cistostomfa
de lograrse mediante (MIR 07-08, 104):
Sondaje vesical.
Talla vesical.
t
No dilatacin
t
Dilatacin unilateral Dilatacin

i
Catter ureteral. bilateral
Nefrostom a.
Estudio mdico
En caso contrario, hay que valorar el grado de sufrimiento renal y su reversi Birreno Monorreno
bilidad. En los casos en que hay destruccin irreversible de la va urinaria, es

L
necesario realizar una derivacin urinaria definitiva (Figura 28) (MIR 08-09,

103). Estudio (UIV, TC)
Fracaso renal

Derivactn de va
PREGUNTAS ./ MIR 1 1 -12, 105
M I R
urinaria superior
./ MIR 08-09, 103 (nefrostoma/doble J)
./ MIR 07-08, 1 04, 1 05
Figura 28. Procedimiento de actuacin frente a la uropata obstructiva

I d e a s c l a v e RS
" El dolor suele estar presente en la obstruccin aguda. Sin embargo,
en la crnica, es frecuente su ausencia.

" La uropata obstructiva puede producir insuficiencia renal si no se " Despus de resolver una uropata obstructiva puede producirse
resuelve a tiempo. una fase de poliuria.

11
U rologa

, ,
D ISFUNCION ERECTI L

sentar DE (MIR 07-08, 1 06). Estn implicados mecanismos vasculares,


Tema sencillo y muy rentable. Se debe
neuropticos y disfuncin gonadal.
incidir en los factores de riesgo,
el tratamiento y sus contraindicaciones. Enfermedad cardiovascular. Cardiopata, hipertensin arterial, enfer
Es conveniente conocer muy bien medad vascular perifrica y descenso del colesterol HDL se han relacio
las preguntas aparecidas con anterioridad. nado de manera clara con la disfuncin erctil.
Tabaquismo. Factor independiente.
Secundaria a frmacos. Aqullos que causan hiperprolactinemia, que
disminuyen los niveles de testosterona, psicotropos y antihipertensivos.
1 0.1 . I ntroduccin Secundaria a consumo de drogas. Cocana, herona . . .
Trastornos afectivos. Depresin.

La disfuncin erctil (DE) se define como la incapacidad persistente o recu


rrente para conseguir o mantener la suficiente rigidez del pene que permita 1 0.5. Diagnstico
una relacin sexual satisfactoria. Debe tener una duracin mnima de 3 meses.

El diagnstico debe basarse en los siguientes componentes:


1 0.2. Prevalencia Historia clnica y sexual. Investigar los posibles factores de riesgo im
plicados.
Exploracin fsica. Encaminada a descartar enfermedad vascular, en
En Estados Unidos, en un estudio en varones de entre 40-70 aos, se estim fermedades neurolgicas, trastornos genitales y endocrinopatas. En
que la prevalencia global era del 52%. La prevalencia en Espaa se estima en varones mayores de 50 aos se incluir tacto rectal.
1 .500.000 a 2.000.000 varones, en torno al 1 2, 1 %. Determinaciones analticas. Glucemia basal, perfil lipdico, testostero
na total y libre y prolactina.
Adems, resulta conveniente solicitar hemograma, funcin renal y he
1 0.3. Etiologa ptica.
Pruebas especializadas. nicamente se precisan en ocasiones muy
seleccionadas.
Se puede clasificar en (MIR 04-05, 108):
Orgnica. Causas vasculares (las ms frecuentes, 60-80%), neurolgicas
(1 0-20%), hormonales (5-1 0%) o locales. 1 0.6. Tratamiento
Psicgena.
Mixta. En la mayora de los casos de etiologa orgnica se aade un
componente psicolgico. Se puede estructurar en tres escalones o etapas, que sern superadas en
funcin de fracaso del escaln previo.

1 0.4. Factores de riesgo Frmacos orales


Citrato de sildenafilo. Se considera actualmente como el tratamiento
Edad. Factor independiente. farmacolgico de eleccin en la DE. Hoy en da existen nuevos frmacos
Diabetes. Es la enfermedad endocrina ms frecuente asociada a dis basados en l, como vardenafilo y tadalafilo. Se trata de un inhibidor de
funcin erctil. Supone una probabilidad tres veces superior de pre- la fosfodiesterasa tipo 5 (PDES). Induce la relajacin del msculo liso del
-
Urologa 1 1O
cuerpo cavernoso, liberando xido ntrico (NO). El NO liberado por el Segunda lnea
endotelio vascular y por las terminaciones nerviosas no adrenrgicas/
no colinrgicas es el principal neurotransmisor de la ereccin. Precisa de Terapia intracavernosa. Alprostadil (PGE1 ), mediante inyeccin directa en
deseo sexual y estimulacin previa para su efecto. los cuerpos cavernosos. Otros frmacos son la papaverina y la fentolamina.
Las contraindicaciones absolutas de sildenafilo son:
Administracin concomitante con nitratos o frmacos donadores Tercera lnea
de xido ntrico por el riesgo de hipotensin grave (dinitrato/mo
nonitrato de isosorbida, molsidomina, nicorandil, nitroglicerina, ni Ciruga de revascularizacin (venosa, arterial). Implante de prtesis
troprusiato sdico) (MIR 06-07, 97; MIR 05-06, 108). de pene.
Pacientes en los que est desaconsejada la actividad sexual (angina
inestable, insuficiencia cardaca o infarto reciente, hace menos de
6 meses). ./ MIR 07-08, 106
./ MIR 06-07, 97
Apomorfina: agonista dopaminrgico que acta a nivel central sobre ./ MIR 05-06, 108
el mecanismo de la ereccin. Est contraindicado en sujetos que ten ./ MIR 04-05, 1 08
gan desaconsejada la actividad sexual.

" El sildenafilo est contraindicado en pacientes que toman nitratos o


I d e a s c l a v e 2'S frmacos donadores de xido ntrico, en pacientes con infarto agu
do de miocardio en los ltimos 6 meses, y en pacientes con insufi
" La causa ms frecuente de disfuncin erctil es vascular. ciencia cardaca grave o angina inestable.

" La enfermedad endocrina ms relacionada con ella es la diabetes


mellitus.
Manual CTO de Medicina y Ciruga, 9. edicin

Bibliografa
Urologa
r, Castieiras Fernndez J. Libro del Residente de Urologa. Madrid. Grficas r, Resel Estvez L, Moreno Sierra J. Tratado de oncologa urolgica. Madrid.
Marte, 2006. Grupo Saned, 2003.
r, European Association of Urology. Guas clnicas Europeas 2013. Arnhem. r, Wein AJ, Kavoussi LR, Novick AC, Partin AW, Peters CA. Campe/1-Walsh
EAU, 2013. Urologa. Argentina. Panamericana, 2008.
r, Jimnez Cruz JF, Rioja Sanz LA. Tratado de urologa. Barcelona. Prous
Science, 2006.

Bibl i o grafa --- -- 11 ---

You might also like